Health Assessment Exam 3

Réussis tes devoirs et examens dès maintenant avec Quizwiz!

52. The nurse is reviewing a patient's medical record and notes that he is in a coma. Using the Glasgow Coma Scale, which number indicates that the patient is in a coma? a. 6 b. 12 c. 15 d. 24

ANS: A A fully alert, normal person has a score of 15, whereas a score of 7 or less reflects coma on the Glasgow Coma Scale (see Figure 23-59). DIF: Cognitive Level: Applying (Application) REF: p. 673 MSC: Client Needs: Physiologic Integrity: Physiologic Adaptation

33. In the assessment of a 1-month-old infant, the nurse notices a lack of response to noise or stimulation. The mother reports that in the last week he has been sleeping all of the time, and when he is awake all he does is cry. The nurse hears that the infant's cries are very high pitched and shrill. What should be the nurse's appropriate response to these findings? a. Refer the infant for further testing. b. Talk with the mother about eating habits. c. Do nothing; these are expected findings for an infant this age. d. Tell the mother to bring the baby back in 1 week for a recheck.

ANS: A A high-pitched, shrill cry or cat-sounding screech occurs with central nervous system damage. Lethargy, hyporeactivity, and hyperirritability, as well as the parent's report of significant changes in behavior all warrant referral. The other options are not correct responses. DIF: Cognitive Level: Analyzing (Analysis) REF: p. 661 MSC: Client Needs: Health Promotion and Maintenance

23. A patient describes feeling an unreasonable, irrational fear of snakes. His fear is so persistent that he can no longer comfortably look at even pictures of snakes and has made an effort to identify all the places he might encounter a snake and avoids them. The nurse recognizes that he: a. Has a snake phobia. b. Is a hypochondriac; snakes are usually harmless. c. Has an obsession with snakes. d. Has a delusion that snakes are harmful, which must stem from an early traumatic incident involving snakes.

ANS: A A phobia is a strong, persistent, irrational fear of an object or situation; the person feels driven to avoid it. (See Table 5-7 for the definitions of the other terms.) PTS: 1 DIF: Cognitive Level: Applying (Application) REF: p. 85 MSC: Client Needs: Psychosocial Integrity

11. A 21-year-old patient has a head injury resulting from trauma and is unconscious. There are no other injuries. During the assessment what would the nurse expect to find when testing the patient's deep tendon reflexes? a. Reflexes will be normal. b. Reflexes cannot be elicited. c. All reflexes will be diminished but present. d. Some reflexes will be present, depending on the area of injury.

ANS: A A reflex is a defense mechanism of the nervous system. It operates below the level of conscious control and permits a quick reaction to potentially painful or damaging situations. DIF: Cognitive Level: Applying (Application) REF: p. 637 MSC: Client Needs: Safe and Effective Care Environment: Management of Care

35. To assess the head control of a 4-month-old infant, the nurse lifts up the infant in a prone position while supporting his chest. The nurse looks for what normal response? The infant: a. Raises the head, and arches the back. b. Extends the arms, and drops down the head. c. Flexes the knees and elbows with the back straight. d. Holds the head at 45 degrees, and keeps the back straight.

ANS: A At 3 months of age, the infant raises the head and arches the back as if in a swan dive. This response is the Landau reflex, which persists until 1 years of age (see Figure 23-43). The other responses are incorrect. DIF: Cognitive Level: Applying (Application) REF: p. 663 MSC: Client Needs: Health Promotion and Maintenance

3. During a speculum inspection of the vagina, the nurse would expect to see what at the end of the vaginal canal? a. Cervix b. Uterus c. Ovaries d. Fallopian tubes

ANS: A At the end of the canal, the uterine cervix projects into the vagina. DIF: Cognitive Level: Remembering (Knowledge) REF: p. 738 MSC: Client Needs: Physiologic Integrity: Physiologic Adaptation

47. The nurse should use which test to check for large amounts of fluid around the patella? a. Ballottement b. Tinel sign c. Phalen test d. McMurray test

ANS: A Ballottement of the patella is reliable when large amounts of fluid are present. The Tinel sign and the Phalen test are used to check for carpal tunnel syndrome. The McMurray test is used to test the knee for a torn meniscus. DIF: Cognitive Level: Understanding (Comprehension) REF: p. 601 MSC: Client Needs: Safe and Effective Care Environment: Management of Care

26. A patient states, "I can hear a crunching or grating sound when I kneel." She also states that "it is very difficult to get out of bed in the morning because of stiffness and pain in my joints." The nurse should assess for signs of what problem? a. Crepitation b. Bone spur c. Loose tendon d. Fluid in the knee joint

ANS: A Crepitation is an audible and palpable crunching or grating that accompanies movement and occurs when articular surfaces in the joints are roughened, as with rheumatoid arthritis. The other options are not correct. DIF: Cognitive Level: Analyzing (Analysis) REF: p. 590 MSC: Client Needs: Physiologic Integrity: Physiologic Adaptation

38. During morning rounds, the nurse asks a patient, "How are you today?" The patient responds, "You today, you today, you today!" and mumbles the words. This speech pattern is an example of: a. Echolalia b. Clanging c. Word salad d. Perseveration

ANS: A Echolalia occurs when a person imitates or repeats another's words or phrases, often with a mumbling, mocking, or a mechanical tone. PTS: 1 DIF: Cognitive Level: Applying (Application) REF: p. 76 MSC: Client Needs: Psychosocial Integrity

44. The nurse is examining a 6-month-old infant and places the infant's feet flat on the table and flexes his knees up. The nurse notes that the right knee is significantly lower than the left. Which of these statements is true of this finding? a. This finding is a positive Allis sign and suggests hip dislocation. b. The infant probably has a dislocated patella on the right knee. c. This finding is a negative Allis sign and normal for an infant of this age. d. The infant should return to the clinic in 2 weeks to see if his condition has changed.

ANS: A Finding one knee significantly lower than the other is a positive Allis sign and suggests hip dislocation. Normally, the tops of the knees are at the same elevation. The other statements are not correct. DIF: Cognitive Level: Analyzing (Analysis) REF: p. 611 MSC: Client Needs: Health Promotion and Maintenance

2. A patient tells the nurse that she is having a hard time bringing her hand to her mouth when she eats or tries to brush her teeth. The nurse knows that for her to move her hand to her mouth, she must perform which movement? a. Flexion b. Abduction c. Adduction d. Extension

ANS: A Flexion, or bending a limb at a joint, is required to move the hand to the mouth. Extension is straightening a limb at a joint. Moving a limb toward the midline of the body is called adduction; abduction is moving a limb away from the midline of the body. DIF: Cognitive Level: Understanding (Comprehension) REF: p. 578 MSC: Client Needs: Physiologic Integrity: Physiologic Adaptation

34. The nurse discovers speech problems in a patient during an assessment. The patient has spontaneous speech, but it is mostly absent or is reduced to a few stereotypical words or sounds. This finding reflects which type of aphasia? a. Global b. Broca's c. Dysphonic d. Wernicke's

ANS: A Global aphasia is the most common and severe form of aphasia. Spontaneous speech is absent or reduced to a few stereotyped words or sounds, and prognosis for language recovery is poor. (Broca's aphasia and Wernicke's aphasia are described in Table 5-4.) Dysphonic aphasia is not a valid condition. PTS: 1 DIF: Cognitive Level: Understanding (Comprehension) REF: p. 80 MSC: Client Needs: Psychosocial Integrity

25. During reporting, the nurse hears that a patient is experiencing hallucinations. Which is an example of a hallucination? a. Man believes that his dead wife is talking to him. b. Woman hears the doorbell ring and goes to answer it, but no one is there. c. Child sees a man standing in his closet. When the lights are turned on, it is only a dry cleaning bag. d. Man believes that the dog has curled up on the bed, but when he gets closer he sees that it is a blanket.

ANS: A Hallucinations are sensory perceptions for which no external stimuli exist. They may strike any sense: visual, auditory, tactile, olfactory, or gustatory. PTS: 1 DIF: Cognitive Level: Remembering (Knowledge) REF: p. 85 MSC: Client Needs: Psychosocial Integrity

46. In a person with an upper motor neuron lesion such as a cerebrovascular accident, which of these physical assessment findings should the nurse expect? a. Hyperreflexia b. Fasciculations c. Loss of muscle tone and flaccidity d. Atrophy and wasting of the muscles

ANS: A Hyperreflexia, diminished or absent superficial reflexes, and increased muscle tone or spasticity can be expected with upper motor neuron lesions. The other options reflect a lesion of lower motor neurons (see Table 23-7). DIF: Cognitive Level: Applying (Application) REF: p. 684 MSC: Client Needs: Physiologic Integrity: Physiologic Adaptation

12. The mother of a 5-year-old girl tells the nurse that she has noticed her daughter "scratching at her bottom a lot the last few days." During the assessment, the nurse finds redness and raised skin in the anal area. This finding most likely indicates: a. Pinworms. b. Chickenpox. c. Constipation. d. Bacterial infection.

ANS: A In children, pinworms are a common cause of intense itching and irritated anal skin. The other options are not correct. DIF: Cognitive Level: Analyzing (Analysis) REF: p. 725 MSC: Client Needs: Physiologic Integrity: Physiologic Adaptation

8. A patient is admitted to the unit after an automobile accident. The nurse begins the mental status examination and finds that the patient has dysarthric speech and is lethargic. The nurse's best approach regarding this examination is to: a. Plan to defer the rest of the mental status examination. b. Skip the language portion of the examination, and proceed onto assessing mood and affect. c. Conduct an in-depth speech evaluation, and defer the mental status examination to another time. d. Proceed with the examination, and assess the patient for suicidal thoughts because dysarthria is often accompanied by severe depression.

ANS: A In the mental status examination, the sequence of steps forms a hierarchy in which the most basic functions (consciousness, language) are assessed first. The first steps must be accurately assessed to ensure validity of the steps that follow. For example, if consciousness is clouded, then the person cannot be expected to have full attention and to cooperate with new learning. If language is impaired, then a subsequent assessment of new learning or abstract reasoning (anything that requires language functioning) can give erroneous conclusions. PTS: 1 DIF: Cognitive Level: Analyzing (Analysis)

36. When assessing a newborn infant's genitalia, the nurse notices that the genitalia are somewhat engorged. The labia majora are swollen, the clitoris looks large, and the hymen is thick. The vaginal opening is difficult to visualize. The infant's mother states that she is worried about the labia being swollen. The nurse should reply: a. "This is a normal finding in newborns and should resolve within a few weeks." b. "This finding could indicate an abnormality and may need to be evaluated by a physician." c. "We will need to have estrogen levels evaluated to ensure that they are within normal limits." d. "We will need to keep close watch over the next few days to see if the genitalia decrease in size."

ANS: A It is normal for a newborn's genitalia to be somewhat engorged. A sanguineous vaginal discharge or leukorrhea is normal during the first few weeks because of the maternal estrogen effect. During the early weeks, the genital engorgement resolves, and the labia minora atrophy and remain small until puberty. DIF: Cognitive Level: Applying (Application) REF: p. 759 MSC: Client Needs: Health Promotion and Maintenance

3. The functional units of the musculoskeletal system are the: a. Joints. b. Bones. c. Muscles. d. Tendons.

ANS: A Joints are the functional units of the musculoskeletal system because they permit the mobility needed to perform the activities of daily living. The skeleton (bones) is the framework of the body. The other options are not correct. DIF: Cognitive Level: Remembering (Knowledge) REF: p. 577 MSC: Client Needs: General

11. During a mental status examination, the nurse wants to assess a patient's affect. The nurse should ask the patient which question? a. "How do you feel today?" b. "Would you please repeat the following words?" c. "Have these medications had any effect on your pain?" d. "Has this pain affected your ability to get dressed by yourself?"

ANS: A Judge mood and affect by body language and facial expression and by directly asking, "How do you feel today?" or "How do you usually feel?" The mood should be appropriate to the person's place and condition and should appropriately change with the topics. PTS: 1 DIF: Cognitive Level: Applying (Application) REF: p. 70 MSC: Client Needs: Psychosocial Integrity

31. During an examination, which tests will the nurse collect to screen for cervical cancer? a. Endocervical specimen, cervical scrape, and vaginal pool b. Endocervical specimen, vaginal pool, and acetic acid wash c. Endocervical specimen, potassium hydroxide (KOH) preparation, and acetic acid wash d. Cervical scrape, acetic acid wash, saline mount ("wet prep")

ANS: A Laboratories may vary in method, but usually the test consists of three specimens: endocervical specimen, cervical scrape, and vaginal pool. The other tests (acetic acid wash, KOH preparation, and saline mount) are used to test for sexually transmitted infections. DIF: Cognitive Level: Understanding (Comprehension) REF: pp. 752-753 MSC: Client Needs: Safe and Effective Care Environment: Management of Care

19. A patient drifts off to sleep when she is not being stimulated. The nurse can easily arouse her by calling her name, but the patient remains drowsy during the conversation. The best description of this patient's level of consciousness would be: a. Lethargic b. Obtunded c. Stuporous d. Semialert

ANS: A Lethargic (or somnolent) is when the person is not fully alert, drifts off to sleep when not stimulated, and can be aroused when called by name in a normal voice but looks drowsy. He or she appropriately responds to questions or commands, but thinking seems slow and fuzzy. He or she is inattentive and loses the train of thought. Spontaneous movements are decreased. (See Table 5-3 for the definitions of the other terms.) PTS: 1 DIF: Cognitive Level: Understanding (Comprehension) REF: p. 79 MSC: Client Needs: Psychosocial Integrity

20. A woman who is 8 months pregnant comments that she has noticed a change in her posture and is having lower back pain. The nurse tells her that during pregnancy, women have a posture shift to compensate for the enlarging fetus. This shift in posture is known as: a. Lordosis. b. Scoliosis. c. Ankylosis. d. Kyphosis.

ANS: A Lordosis compensates for the enlarging fetus, which would shift the center of balance forward. This shift in balance, in turn, creates a strain on the low back muscles, felt as low back pain during late pregnancy by some women. Scoliosis is lateral curvature of portions of the spine; ankylosis is extreme flexion of the wrist, as observed with severe rheumatoid arthritis; and kyphosis is an enhanced thoracic curvature of the spine. DIF: Cognitive Level: Understanding (Comprehension) REF: p. 584 MSC: Client Needs: Health Promotion and Maintenance

10. A woman is in the clinic for an annual gynecologic examination. The nurse should plan to begin the interview with the: a. Menstrual history, because it is generally nonthreatening. b. Obstetric history, because it includes the most important information. c. Urinary system history, because problems may develop in this area as well. d. Sexual history, because discussing it first will build rapport.

ANS: A Menstrual history is usually nonthreatening and therefore a good topic with which to begin the interview. Obstetric, urinary, and sexual histories are also part of the interview but not necessarily the best topics with which to start. DIF: Cognitive Level: Applying (Application) REF: p. 740 MSC: Client Needs: Safe and Effective Care Environment: Management of Care

37. To test for gross motor skill and coordination of a 6-year-old child, which of these techniques would be appropriate? Ask the child to: a. Hop on one foot. b. Stand on his head. c. Touch his finger to his nose. d. Make "funny" faces at the nurse.

ANS: A Normally, a child can hop on one foot and can balance on one foot for approximately 5 seconds by 4 years of age and can balance on one foot for 8 to 10 seconds at 5 years of age. Children enjoy performing these tests. Failure to hop after 5 years of age indicates incoordination of gross motor skills. Asking the child to touch his or her finger to the nose checks fine motor coordination; and asking the child to make "funny" faces tests CN VII. Asking a child to stand on his or her head is not appropriate. DIF: Cognitive Level: Applying (Application) REF: p. 668 MSC: Client Needs: Health Promotion and Maintenance

6. A woman brings her husband to the clinic for an examination. She is particularly worried because after a recent fall, he seems to have lost a great deal of his memory of recent events. Which statement reflects the nurse's best course of action? a. Perform a complete mental status examination. b. Refer him to a psychometrician. c. Plan to integrate the mental status examination into the history and physical examination. d. Reassure his wife that memory loss after a physical shock is normal and will soon subside.

ANS: A Performing a complete mental status examination is necessary when any abnormality in affect or behavior is discovered or when family members are concerned about a person's behavioral changes (e.g., memory loss, inappropriate social interaction) or after trauma, such as a head injury. PTS: 1 DIF: Cognitive Level: Applying (Application) REF: p. 69 MSC: Client Needs: Psychosocial Integrity

23. The nurse is preparing to interview a postmenopausal woman. Which of these statements is true as it applies to obtaining the health history of a postmenopausal woman? a. The nurse should ask a postmenopausal woman if she has ever had vaginal bleeding. b. Once a woman reaches menopause, the nurse does not need to ask any history questions. c. The nurse should screen for monthly breast tenderness. d. Postmenopausal women are not at risk for contracting STIs; therefore, these questions can be omitted.

ANS: A Postmenopausal bleeding warrants further workup and referral. The other statements are not true. DIF: Cognitive Level: Understanding (Comprehension) REF: p. 744 MSC: Client Needs: Physiologic Integrity: Reduction of Risk Potential

17. While obtaining a health history of a 3-month-old infant from the mother, the nurse asks about the infant's ability to suck and grasp the mother's finger. What is the nurse assessing? a. Reflexes b. Intelligence c. CNs d. Cerebral cortex function

ANS: A Questions regarding reflexes include such questions as, "What have you noticed about the infant's behavior," "Are the infant's sucking and swallowing seem coordinated," and "Does the infant grasp your finger?" The other responses are incorrect. DIF: Cognitive Level: Understanding (Comprehension) REF: p. 642 MSC: Client Needs: Health Promotion and Maintenance

38. During the assessment of an 80-year-old patient, the nurse notices that his hands show tremors when he reaches for something and his head is always nodding. No associated rigidity is observed with movement. Which of these statements is most accurate? a. These findings are normal, resulting from aging. b. These findings could be related to hyperthyroidism. c. These findings are the result of Parkinson disease. d. This patient should be evaluated for a cerebellar lesion.

ANS: A Senile tremors occasionally occur. These benign tremors include an intention tremor of the hands, head nodding (as if saying yes or no), and tongue protrusion. Tremors associated with Parkinson disease include rigidity, slowness, and a weakness of voluntary movement. The other responses are incorrect. DIF: Cognitive Level: Analyzing (Analysis) REF: p. 669 MSC: Client Needs: Health Promotion and Maintenance

2. The nurse is assessing the mental status of a child. Which statement about children and mental status is true? a. All aspects of mental status in children are interdependent. b. Children are highly labile and unstable until the age of 2 years. c. Children's mental status is largely a function of their parents' level of functioning until the age of 7 years. d. A child's mental status is impossible to assess until the child develops the ability to concentrate.

ANS: A Separating and tracing the development of only one aspect of mental status is difficult. All aspects are interdependent. For example, consciousness is rudimentary at birth because the cerebral cortex is not yet developed. The infant cannot distinguish the self from the mother's body. The other statements are not true. PTS: 1 DIF: Cognitive Level: Understanding (Comprehension) REF: p. 68 MSC: Client Needs: Psychosocial Integrity

39. When performing an external genitalia examination of a 10-year-old girl, the nurse notices that no pubic hair has grown in and the mons and the labia are covered with fine vellus hair. These findings are consistent with stage _____ of sexual maturity, according to the Sexual Maturity Rating scale. a. 1 b. 2 c. 3 d. 4

ANS: A Sexual Maturity Rating stage 1 is the preadolescent stage. There is no pubic hair, and the mons and labia are covered with fine, vellus hair as on the abdomen (see Table 26-1). DIF: Cognitive Level: Applying (Application) REF: p. 739 MSC: Client Needs: Health Promotion and Maintenance

29. The nurse is testing the deep tendon reflexes of a 30-year-old woman who is in the clinic for an annual physical examination. When striking the Achilles heel and quadriceps muscle, the nurse is unable to elicit a reflex. The nurse's next response should be to: a. Ask the patient to lock her fingers and pull. b. Complete the examination, and then test these reflexes again. c. Refer the patient to a specialist for further testing. d. Document these reflexes as 0 on a scale of 0 to 4+.

ANS: A Sometimes the reflex response fails to appear. Documenting the reflexes as absent is inappropriate this soon in the examination. The nurse should try to further encourage relaxation, varying the person's position or increasing the strength of the blow. Reinforcement is another technique to relax the muscles and enhance the response. The person should be asked to perform an isometric exercise in a muscle group somewhat away from the one being tested. For example, to enhance a patellar reflex, the person should be asked to lock the fingers together and pull. DIF: Cognitive Level: Applying (Application) REF: p. 656 MSC: Client Needs: Health Promotion and Maintenance

37. The nurse is administering a Mini-Cog test to an older adult woman. When asked to draw a clock showing the time of 10:45, the patient drew a clock with the numbers out of order and with an incorrect time. This result indicates which finding? a. Cognitive impairment b. Amnesia c. Delirium d. Attention-deficit disorder

ANS: A The Mini-Cog is a newer instrument that screens for cognitive impairment, often found with dementia. The result of an abnormal drawing of a clock and time indicates a cognitive impairment. PTS: 1 DIF: Cognitive Level: Analyzing (Analysis) REF: p. 85 MSC: Client Needs: Psychosocial Integrity

5. An 11-year-old girl is in the clinic for a sports physical examination. The nurse notices that she has begun to develop breasts, and during the conversation the girl reveals that she is worried about her development. The nurse should use which of these techniques to best assist the young girl in understanding the expected sequence for development? The nurse should: a. Use the Tanner scale on the five stages of sexual development. b. Describe her development and compare it with that of other girls her age. c. Use the Jacobsen table on expected development on the basis of height and weight data. d. Reassure her that her development is within normal limits and tell her not to worry about the next step.

ANS: A The Tanner scale on the five stages of pubic hair development is helpful in teaching girls the expected sequence of sexual development (see Table 26-1). The other responses are not appropriate. DIF: Cognitive Level: Applying (Application) REF: p. 739 MSC: Client Needs: Health Promotion and Maintenance

18. The ankle joint is the articulation of the tibia, fibula, and: a. Talus. b. Cuboid. c. Calcaneus. d. Cuneiform bones.

ANS: A The ankle or tibiotalar joint is the articulation of the tibia, fibula, and talus. The other bones listed are foot bones and not part of the ankle joint. DIF: Cognitive Level: Remembering (Knowledge) REF: p. 583 MSC: Client Needs: General

49. The nurse knows that determining whether a person is oriented to his or her surroundings will test the functioning of which structure(s)? a. Cerebrum b. Cerebellum c. CNs d. Medulla oblongata

ANS: A The cerebral cortex is responsible for thought, memory, reasoning, sensation, and voluntary movement. The other structures are not responsible for a person's level of consciousness. DIF: Cognitive Level: Understanding (Comprehension) REF: p. 670 MSC: Client Needs: Safe and Effective Care Environment: Management of Care

6. The nurse is caring for a newborn infant. Thirty hours after birth, the infant passes a dark green meconium stool. The nurse recognizes this is important because the: a. Stool indicates anal patency. b. Dark green color indicates occult blood in the stool. c. Meconium stool can be reflective of distress in the newborn. d. Newborn should have passed the first stool within 12 hours after birth.

ANS: A The first stool passed by the newborn is dark green meconium and occurs within 24 to 48 hours of birth, indicating anal patency. The other responses are not correct. DIF: Cognitive Level: Understanding (Comprehension) REF: p. 723 MSC: Client Needs: Health Promotion and Maintenance

2. The wife of a 65-year-old man tells the nurse that she is concerned because she has noticed a change in her husband's personality and ability to understand. He also cries very easily and becomes angry. The nurse recalls that the cerebral lobe responsible for these behaviors is the __________ lobe. a. Frontal b. Parietal c. Occipital d. Temporal

ANS: A The frontal lobe has areas responsible for personality, behavior, emotions, and intellectual function. The parietal lobe has areas responsible for sensation; the occipital lobe is responsible for visual reception; and the temporal lobe is responsible for hearing, taste, and smell. DIF: Cognitive Level: Understanding (Comprehension) REF: p. 633 MSC: Client Needs: Physiologic Integrity: Physiologic Adaptation

14. The nurse is checking the range of motion in a patient's knee and knows that the knee is capable of which movement(s)? a. Flexion and extension b. Supination and pronation c. Circumduction d. Inversion and eversion

ANS: A The knee is a hinge joint, permitting flexion and extension of the lower leg on a single plane. The knee is not capable of the other movements listed. DIF: Cognitive Level: Understanding (Comprehension) REF: p. 583 MSC: Client Needs: Physiologic Integrity: Physiologic Adaptation

2. During an examination, the nurse observes a female patient's vestibule and expects to see the: a. Urethral meatus and vaginal orifice. b. Vaginal orifice and vestibular (Bartholin) glands. c. Urethral meatus and paraurethral (Skene) glands. d. Paraurethral (Skene) and vestibular (Bartholin) glands.

ANS: A The labial structures encircle a boat-shaped space, or cleft, termed the vestibule. Within the vestibule are numerous openings. The urethral meatus and vaginal orifice are visible. The ducts of the paraurethral (Skene) glands and the vestibular (Bartholin) glands are present but not visible. DIF: Cognitive Level: Understanding (Comprehension) REF: p. 737 MSC: Client Needs: Physiologic Integrity: Physiologic Adaptation

50. When performing a musculoskeletal assessment, the nurse knows that the correct approach for the examination should be: a. Proximal to distal. b. Distal to proximal. c. Posterior to anterior. d. Anterior to posterior.

ANS: A The musculoskeletal assessment should be performed in an orderly approach, head to toe, proximal to distal, from the midline outward. The other options are not correct. DIF: Cognitive Level: Applying (Application) REF: p. 589 MSC: Client Needs: Safe and Effective Care Environment: Management of Care

43. During an assessment of a 62-year-old man, the nurse notices the patient has a stooped posture, shuffling walk with short steps, flat facial expression, and pill-rolling finger movements. These findings would be consistent with: a. Parkinsonism. b. Cerebral palsy. c. Cerebellar ataxia. d. Muscular dystrophy.

ANS: A The stooped posture, shuffling walk, short steps, flat facial expression, and pill-rolling finger movements are all found in parkinsonism. (See Table 23-8 for more information and for the descriptions of the other options.) DIF: Cognitive Level: Analyzing (Analysis) REF: p. 685 MSC: Client Needs: Physiologic Integrity: Physiologic Adaptation

4. The uterus is usually positioned tilting forward and superior to the bladder. This position is known as: a. Anteverted and anteflexed. b. Retroverted and anteflexed. c. Retroverted and retroflexed. d. Superiorverted and anteflexed.

ANS: A The uterus is freely movable, not fixed, and usually tilts forward and superior to the bladder (a position labeled as anteverted and anteflexed). DIF: Cognitive Level: Remembering (Knowledge) REF: p. 738 MSC: Client Needs: General

8. The nurse is reviewing the changes that occur with menopause. Which changes are associated with menopause? a. Uterine and ovarian atrophy, along with a thinning of the vaginal epithelium b. Ovarian atrophy, increased vaginal secretions, and increasing clitoral size c. Cervical hypertrophy, ovarian atrophy, and increased acidity of vaginal secretions d. Vaginal mucosa fragility, increased acidity of vaginal secretions, and uterine hypertrophy

ANS: A The uterus shrinks because of its decreased myometrium. The ovaries atrophy to 1 to 2 cm and are not palpable after menopause. The sacral ligaments relax, and the pelvic musculature weakens; consequently, the uterus droops. The cervix shrinks and looks paler with a thick glistening epithelium. The vaginal epithelium atrophies, becoming thinner, drier, and itchy. The vaginal pH becomes more alkaline, and secretions are decreased, which results in a fragile mucosal surface that is at risk for vaginitis. DIF: Cognitive Level: Understanding (Comprehension) REF: p. 740 MSC: Client Needs: Health Promotion and Maintenance

37. During a vaginal examination of a 38-year-old woman, the nurse notices that the vulva and vagina are erythematous and edematous with thick, white, curdlike discharge adhering to the vaginal walls. The woman reports intense pruritus and thick white discharge from her vagina. The nurse knows that these history and physical examination findings are most consistent with which condition? a. Candidiasis b. Trichomoniasis c. Atrophic vaginitis d. Bacterial vaginosis

ANS: A The woman with candidiasis often reports intense pruritus and thick white discharge. The vulva and vagina are erythematous and edematous. The discharge is usually thick, white, and curdlike. Infection with trichomoniasis causes a profuse, watery, gray-green, and frothy discharge. Bacterial vaginosis causes a profuse discharge that has a "foul, fishy, rotten" odor. Atrophic vaginitis may have a mucoid discharge. (See Table 26-5 for complete descriptions of each option.) DIF: Cognitive Level: Analyzing (Analysis) REF: p. 768 MSC: Client Needs: Physiologic Integrity: Physiologic Adaptation

9. Of the 33 vertebrae in the spinal column, there are: a. 5 lumbar. b. 5 thoracic. c. 7 sacral. d. 12 cervical.

ANS: A There are 7 cervical, 12 thoracic, 5 lumbar, 5 sacral, and 3 to 4 coccygeal vertebrae in the spinal column. DIF: Cognitive Level: Remembering (Knowledge) REF: p. 579

34. Which of these tests would the nurse use to check the motor coordination of an 11-month-old infant? a. Denver II b. Stereognosis c. Deep tendon reflexes d. Rapid alternating movements

ANS: A To screen gross and fine motor coordination, the nurse should use the Denver II with its age-specific developmental milestones. Stereognosis tests a person's ability to recognize objects by feeling them and is not appropriate for an 11-month-old infant. Testing the deep tendon reflexes is not appropriate for checking motor coordination. Testing rapid alternating movements is appropriate for testing coordination in adults. DIF: Cognitive Level: Understanding (Comprehension) REF: p. 662 MSC: Client Needs: Health Promotion and Maintenance

15. During the taking of the health history, a patient tells the nurse that "it feels like the room is spinning around me." The nurse would document this finding as: a. Vertigo. b. Syncope. c. Dizziness. d. Seizure activity.

ANS: A True vertigo is rotational spinning caused by a neurologic dysfunction or a problem in the vestibular apparatus or the vestibular nuclei in the brainstem. Syncope is a sudden loss of strength or a temporary loss of consciousness. Dizziness is a lightheaded, swimming sensation. Seizure activity is characterized by altered or loss of consciousness, involuntary muscle movements, and sensory disturbances. DIF: Cognitive Level: Applying (Application) REF: p. 641 MSC: Client Needs: Physiologic Integrity: Physiologic Adaptation

47. A 59-year-old patient has a herniated intervertebral disk. Which of the following findings should the nurse expect to see on physical assessment of this individual? a. Hyporeflexia b. Increased muscle tone c. Positive Babinski sign d. Presence of pathologic reflexes

ANS: A With a herniated intervertebral disk or lower motor neuron lesion, loss of tone, flaccidity, atrophy, fasciculations, and hyporeflexia or areflexia are demonstrated. No Babinski sign or pathologic reflexes would be observed (see Table 23-7). The other options reflect a lesion of upper motor neurons. DIF: Cognitive Level: Applying (Application) REF: p. 655 MSC: Client Needs: Physiologic Integrity: Physiologic Adaptation

1. The nurse is assessing a patient who is admitted with possible delirium. Which of these are manifestations of delirium? Select all that apply. a. Develops over a short period. b. Person is experiencing apraxia. c. Person is exhibiting memory impairment or deficits. d. Occurs as a result of a medical condition, such as systemic infection. e. Person is experiencing agnosia.

ANS: A, C, D Delirium is a disturbance of consciousness that develops over a short period and may be attributable to a medical condition. Memory deficits may also occur. Apraxia and agnosia occur with dementia. PTS: 1 DIF: Cognitive Level: Applying (Application) REF: p. 83 MSC: Client Needs: Psychosocial Integrity

14. During a mental status assessment, which question by the nurse would best assess a person's judgment? a. "Do you feel that you are being watched, followed, or controlled?" b. "Tell me what you plan to do once you are discharged from the hospital." c. "What does the statement, 'People in glass houses shouldn't throw stones,' mean to you?" d. "What would you do if you found a stamped, addressed envelope lying on the sidewalk?"

ANS: B A person exercises judgment when he or she can compare and evaluate the alternatives in a situation and reach an appropriate course of action. Rather than testing the person's response to a hypothetical situation (as illustrated in the option with the envelope), the nurse should be more interested in the person's judgment about daily or long-term goals, the likelihood of acting in response to delusions or hallucinations, and the capacity for violent or suicidal behavior. PTS: 1 DIF: Cognitive Level: Applying (Application) REF: p. 74 MSC: Client Needs: Psychosocial Integrity

22. A patient has been diagnosed with osteoporosis and asks the nurse, "What is osteoporosis?" The nurse explains that osteoporosis is defined as: a. Increased bone matrix. b. Loss of bone density. c. New, weaker bone growth. d. Increased phagocytic activity.

ANS: B After age 40 years, a loss of bone matrix (resorption) occurs more rapidly than new bone formation. The net effect is a gradual loss of bone density, or osteoporosis. The other options are not correct. DIF: Cognitive Level: Remembering (Knowledge) REF: p. 584 MSC: Client Needs: Physiologic Integrity: Physiologic Adaptation

10. An imaginary line connecting the highest point on each iliac crest would cross the __________ vertebra. a. First sacral b. Fourth lumbar c. Seventh cervical d. Twelfth thoracic

ANS: B An imaginary line connecting the highest point on each iliac crest crosses the fourth lumbar vertebra. The other options are not correct. DIF: Cognitive Level: Remembering (Knowledge) REF: p. 579 MSC: Client Needs: General

26. The assessment of a 60-year-old patient has taken longer than anticipated. In testing his pain perception, the nurse decides to complete the test as quickly as possible. When the nurse applies the sharp point of the pin on his arm several times, he is only able to identify these as one "very sharp prick." What would be the most accurate explanation for this? a. The patient has hyperesthesia as a result of the aging process. b. This response is most likely the result of the summation effect. c. The nurse was probably not poking hard enough with the pin in the other areas. d. The patient most likely has analgesia in some areas of arm and hyperalgesia in others.

ANS: B At least 2 seconds should be allowed to elapse between each stimulus to avoid summation. With summation, frequent consecutive stimuli are perceived as one strong stimulus. The other responses are incorrect. DIF: Cognitive Level: Analyzing (Analysis) REF: p. 652 MSC: Client Needs: Safe and Effective Care Environment: Management of Care

35. A patient repeats, "I feel hot. Hot, cot, rot, tot, got. I'm a spot." The nurse documents this as an illustration of: a. Blocking b. Clanging c. Echolalia d. Neologism

ANS: B Clanging is word choice based on sound, not meaning, and includes nonsense rhymes and puns. (See Table 5-6 for the definitions of the other terms.) PTS: 1 DIF: Cognitive Level: Understanding (Comprehension) REF: p. 80 MSC: Client Needs: Psychosocial Integrity

36. A 40-year-old man has come into the clinic with complaints of extreme pain in his toes. The nurse notices that his toes are slightly swollen, reddened, and warm to the touch. His complaints would suggest: a. Osteoporosis. b. Acute gout. c. Ankylosing spondylitis. d. Degenerative joint disease.

ANS: B Clinical findings for acute gout consist of redness, swelling, heat, and extreme pain like a continuous throbbing. Gout is a metabolic disorder of disturbed purine metabolism, associated with elevated serum uric acid. (See Table 22-1 for descriptions of the other terms.) DIF: Cognitive Level: Analyzing (Analysis) REF: p. 627 MSC: Client Needs: Physiologic Integrity: Physiologic Adaptation

7. The ability that humans have to perform very skilled movements such as writing is controlled by the: a. Basal ganglia. b. Corticospinal tract. c. Spinothalamic tract. d. Extrapyramidal tract.

ANS: B Corticospinal fibers mediate voluntary movement, particularly very skilled, discrete, and purposeful movements, such as writing. The corticospinal tract, also known as the pyramidal tract, is a newer, "higher" motor system that humans have that permits very skilled and purposeful movements. The other responses are not related to skilled movements. DIF: Cognitive Level: Understanding (Comprehension) REF: p. 636 MSC: Client Needs: General

9. A 54-year-old woman who has just completed menopause is in the clinic today for a yearly physical examination. Which of these statements should the nurse include in patient education? "A postmenopausal woman: a. Is not at any greater risk for heart disease than a younger woman." b. Should be aware that she is at increased risk for dyspareunia because of decreased vaginal secretions." c. Has only stopped menstruating; there really are no other significant changes with which she should be concerned." d. Is likely to have difficulty with sexual pleasure as a result of drastic changes in the female sexual response cycle."

ANS: B Decreased vaginal secretions leave the vagina dry and at risk for irritation and pain with intercourse (dyspareunia). The other statements are incorrect. DIF: Cognitive Level: Applying (Application) REF: p. 740 MSC: Client Needs: Health Promotion and Maintenance

54. The nurse is assessing the neurologic status of a patient who has a late-stage brain tumor. With the reflex hammer, the nurse draws a light stroke up the lateral side of the sole of the foot and inward, across the ball of the foot. In response, the patient's toes fan out, and the big toe shows dorsiflexion. The nurse interprets this result as: a. Negative Babinski sign, which is normal for adults. b. Positive Babinski sign, which is abnormal for adults. c. Clonus, which is a hyperactive response. d. Achilles reflex, which is an expected response.

ANS: B Dorsiflexion of the big toe and fanning of all toes is a positive Babinski sign, also called up-going toes. This response occurs with upper motor neuron disease of the corticospinal (or pyramidal) tract and is an abnormal finding for adults. DIF: Cognitive Level: Analyzing (Analysis) REF: p. 660 MSC: Client Needs: Physiologic Integrity: Physiologic Adaptation

11. After completing an assessment of a 60-year-old man with a family history of colon cancer, the nurse discusses with him early detection measures for colon cancer. The nurse should mention the need for a(n): a. Annual proctoscopy. b. Colonoscopy every 10 years. c. Fecal test for blood every 6 months. d. DREs every 2 years.

ANS: B Early detection measures for colon cancer include a DRE performed annually after age 50 years, an annual fecal occult blood test after age 50 years, a sigmoidoscopic examination every 5 years or a colonoscopy every 10 years after age 50 years, and a PSA blood test annually for men over 50 years old, except beginning at age 45 years for black men (American Cancer Society, 2006).

50. During an examination, the nurse notices severe nystagmus in both eyes of a patient. Which conclusion by the nurse is correct? Severe nystagmus in both eyes: a. Is a normal occurrence. b. May indicate disease of the cerebellum or brainstem. c. Is a sign that the patient is nervous about the examination. d. Indicates a visual problem, and a referral to an ophthalmologist is indicated.

ANS: B End-point nystagmus at an extreme lateral gaze normally occurs; however, the nurse should carefully assess any other nystagmuses. Severe nystagmus occurs with disease of the vestibular system, cerebellum, or brainstem. DIF: Cognitive Level: Analyzing (Analysis) REF: p. 645 MSC: Client Needs: Health Promotion and Maintenance

39. A woman who has had rheumatoid arthritis for years is starting to notice that her fingers are drifting to the side. The nurse knows that this condition is commonly referred to as: a. Radial drift. b. Ulnar deviation. c. Swan-neck deformity. d. Dupuytren contracture.

ANS: B Fingers drift to the ulnar side because of stretching of the articular capsule and muscle imbalance caused by chronic rheumatoid arthritis. A radial drift is not observed. (See Table 22-4 for descriptions of swan-neck deformity and Dupuytren contracture.) DIF: Cognitive Level: Applying (Application) REF: p. 624 MSC: Client Needs: Physiologic Integrity: Physiologic Adaptation

35. The nurse is preparing to examine the external genitalia of a school-age girl. Which position would be most appropriate in this situation? a. In the parent's lap b. In a frog-leg position on the examining table c. In the lithotomy position with the feet in stirrups d. Lying flat on the examining table with legs extended

ANS: B For school-age children, placing them on the examining table in a frog-leg position is best. With toddlers and preschoolers, having the child on the parent's lap in a frog-leg position is best. DIF: Cognitive Level: Applying (Application) REF: p. 759 MSC: Client Needs: Health Promotion and Maintenance

41. A patient's annual physical examination reveals a lateral curvature of the thoracic and lumbar segments of his spine; however, this curvature disappears with forward bending. The nurse knows that this abnormality of the spine is called: a. Structural scoliosis. b. Functional scoliosis. c. Herniated nucleus pulposus. d. Dislocated hip.

ANS: B Functional scoliosis is flexible and apparent with standing but disappears with forward bending. Structural scoliosis is fixed; the curvature shows both when standing and when bending forward. (See Table 22-7 for description of herniated nucleus pulposus.) These findings are not indicative of a dislocated hip. DIF: Cognitive Level: Analyzing (Analysis) REF: p. 629 MSC: Client Needs: Physiologic Integrity: Physiologic Adaptation

48. A patient tells the nurse that, "All my life I've been called 'knock knees'." The nurse knows that another term for knock knees is: a. Genu varum. b. Genu valgum. c. Pes planus. d. Metatarsus adductus.

ANS: B Genu valgum is also known as knock knees and is present when more than 2.5 cm is between the medial malleoli when the knees are together. DIF: Cognitive Level: Understanding (Comprehension) REF: p. 601 MSC: Client Needs: Physiologic Integrity: Physiologic Adaptation

45. A woman has just been diagnosed with HPV or genital warts. The nurse should counsel her to receive regular examinations because this virus makes her at a higher risk for _______ cancer. a. Uterine b. Cervical c. Ovarian d. Endometrial

ANS: B HPV is the virus responsible for most cases of cervical cancer, not the other options. DIF: Cognitive Level: Applying (Application) REF: p. 761 MSC: Client Needs: Health Promotion and Maintenance

9. A 30-year-old woman is visiting the clinic because of "pain in my bottom when I have a bowel movement." The nurse should assess for which problem? a. Pinworms b. Hemorrhoids c. Colon cancer d. Fecal incontinence

ANS: B Having painful bowel movements, known as dyschezia, may be attributable to a local condition (hemorrhoid or fissure) or constipation. The other responses are not correct. DIF: Cognitive Level: Applying (Application) REF: p. 723 MSC: Client Needs: Physiologic Integrity: Physiologic Adaptation

41. During an assessment of a 22-year-old woman who sustained a head injury from an automobile accident 4 hours earlier, the nurse notices the following changes: pupils were equal, but now the right pupil is fully dilated and nonreactive, and the left pupil is 4 mm and reacts to light. What do these findings suggest? a. Injury to the right eye b. Increased intracranial pressure c. Test inaccurately performed d. Normal response after a head injury

ANS: B In a person with a brain injury, a sudden, unilateral, dilated, and nonreactive pupil is ominous. CN III runs parallel to the brainstem. When increasing intracranial pressure pushes down the brainstem (uncal herniation), it puts pressure on CN III, causing pupil dilation. The other responses are incorrect. DIF: Cognitive Level: Analyzing (Analysis) REF: p. 671 MSC: Client Needs: Physiologic Integrity: Physiologic Adaptation

4. The structure that secretes a thin, milky alkaline fluid to enhance the viability of sperm is the: a. Cowper gland. b. Prostate gland. c. Median sulcus. d. Bulbourethral gland.

ANS: B In men, the prostate gland secretes a thin milky alkaline fluid that enhances sperm viability. The Cowper glands (also known as bulbourethral glands) secrete a clear, viscid mucus. The median sulcus is a groove that divides the lobes of the prostate gland and does not secrete fluid. DIF: Cognitive Level: Remembering (Knowledge) REF: p. 722 MSC: Client Needs: General

20. A nurse is assessing a patient's risk of contracting a sexually transmitted infection (STI). An appropriate question to ask would be: a. "You know that it's important to use condoms for protection, right?" b. "Do you use a condom with each episode of sexual intercourse?" c. "Do you have a sexually transmitted infection?" d. "You are aware of the dangers of unprotected sex, aren't you?"

ANS: B In reviewing a patient's risk for STIs, the nurse should ask in a nonconfrontational manner whether condoms are being used during each episode of sexual intercourse. Asking a person whether he or she has an infection does not address the risk. DIF: Cognitive Level: Understanding (Comprehension) REF: p. 743 MSC: Client Needs: Physiologic Integrity: Reduction of Risk Potential

19. A married couple has come to the clinic seeking advice on pregnancy. They have been trying to conceive for 4 months and have not been successful. What should the nurse do first? a. Ascertain whether either of them has been using broad-spectrum antibiotics. b. Explain that couples are considered infertile after 1 year of unprotected intercourse. c. Immediately refer the woman to an expert in pelvic inflammatory disease—the most common cause of infertility. d. Explain that couples are considered infertile after 3 months of engaging in unprotected intercourse and that they will need a referral to a fertility expert.

ANS: B Infertility is considered after 1 year of engaging in unprotected sexual intercourse without conceiving. The other actions are not appropriate. DIF: Cognitive Level: Applying (Application) REF: p. 743 MSC: Client Needs: Psychosocial Integrity

51. The nurse knows that testing kinesthesia is a test of a person's: a. Fine touch. b. Position sense. c. Motor coordination. d. Perception of vibration.

ANS: B Kinesthesia, or position sense, is the person's ability to perceive passive movements of the extremities. The other options are incorrect. DIF: Cognitive Level: Understanding (Comprehension) REF: p. 653 MSC: Client Needs: Safe and Effective Care Environment: Management of Care

30. An 80-year-old woman is visiting the clinic for a checkup. She states, "I can't walk as much as I used to." The nurse is observing for motor dysfunction in her hip and should ask her to: a. Internally rotate her hip while she is sitting. b. Abduct her hip while she is lying on her back. c. Adduct her hip while she is lying on her back. d. Externally rotate her hip while she is standing.

ANS: B Limited abduction of the hip while supine is the most common motion dysfunction found in hip disease. The other options are not correct. DIF: Cognitive Level: Analyzing (Analysis) REF: p. 600 MSC: Client Needs: Physiologic Integrity: Physiologic Adaptation

26. The nurse has just completed an inspection of a nulliparous woman's external genitalia. Which of these would be a description of a finding within normal limits? a. Redness of the labia majora b. Multiple nontender sebaceous cysts c. Discharge that is foul smelling and irritating d. Gaping and slightly shriveled labia majora

ANS: B No lesions should be noted, except for the occasional sebaceous cysts, which are yellowish 1-cm nodules that are firm, nontender, and often multiple. The labia majora are dark pink, moist, and symmetric; redness indicates inflammation or lesions. Discharge that is foul smelling and irritating may indicate infection. In the nulliparous woman, the labia majora meet in the midline, are symmetric and plump. DIF: Cognitive Level: Applying (Application) REF: p. 747 MSC: Client Needs: Safe and Effective Care Environment: Management of Care

33. The nurse is palpating a female patient's adnexa. The findings include a firm, smooth uterine wall; the ovaries are palpable and feel smooth and firm. The fallopian tube is firm and pulsating. The nurse's most appropriate course of action would be to: a. Tell the patient that her examination is normal. b. Give her an immediate referral to a gynecologist. c. Suggest that she return in a month for a recheck to verify the findings. d. Tell the patient that she may have an ovarian cyst that should be evaluated further.

ANS: B Normally, the uterine wall feels firm and smooth, with the contour of the fundus rounded. Ovaries are not often palpable, but when they are, they normally feel smooth, firm, and almond shaped and are highly movable, sliding through the fingers. The fallopian tube is not normally palpable. No other mass or pulsation should be felt. Pulsation or palpable fallopian tube suggests ectopic pregnancy, which warrants immediate referral. DIF: Cognitive Level: Applying (Application) REF: p. 757 MSC: Client Needs: Health Promotion and Maintenance

21. An 85-year-old patient comments during his annual physical examination that he seems to be getting shorter as he ages. The nurse should explain that decreased height occurs with aging because: a. Long bones tend to shorten with age. b. The vertebral column shortens. c. A significant loss of subcutaneous fat occurs. d. A thickening of the intervertebral disks develops.

ANS: B Postural changes are evident with aging; decreased height is most noticeable and is due to shortening of the vertebral column. Long bones do not shorten with age. Intervertebral disks actually get thinner with age. Subcutaneous fat is not lost but is redistributed to the abdomen and hips. DIF: Cognitive Level: Applying (Application) REF: pp. 584-585 MSC: Client Needs: Health Promotion and Maintenance

36. During an interview, the nurse notes that the patient gets up several times to wash her hands even though they are not dirty. This behavior is an example of: a. Social phobia b. Compulsive disorder c. Generalized anxiety disorder d. Posttraumatic stress disorder

ANS: B Repetitive behaviors, such as handwashing, are behaviors that the person feels driven to perform in response to an obsession. The behaviors are aimed at preventing or reducing distress or preventing some dreaded event or situation. PTS: 1 DIF: Cognitive Level: Understanding (Comprehension) REF: p. 84 MSC: Client Needs: Psychosocial Integrity

27. A patient is able to flex his right arm forward without difficulty or pain but is unable to abduct his arm because of pain and muscle spasms. The nurse should suspect: a. Crepitation. b. Rotator cuff lesions. c. Dislocated shoulder. d. Rheumatoid arthritis.

ANS: B Rotator cuff lesions may limit range of motion and cause pain and muscle spasms during abduction, whereas forward flexion remains fairly normal. The other options are not correct. DIF: Cognitive Level: Analyzing (Analysis) REF: p. 594 MSC: Client Needs: Physiologic Integrity: Physiologic Adaptation

18. In obtaining a health history on a 74-year-old patient, the nurse notes that he drinks alcohol daily and that he has noticed a tremor in his hands that affects his ability to hold things. With this information, what response should the nurse make? a. "Does your family know you are drinking every day?" b. "Does the tremor change when you drink alcohol?" c. "We'll do some tests to see what is causing the tremor." d. "You really shouldn't drink so much alcohol; it may be causing your tremor."

ANS: B Senile tremor is relieved by alcohol, although not a recommended treatment. The nurse should assess whether the person is abusing alcohol in an effort to relieve the tremor. DIF: Cognitive Level: Analyzing (Analysis) REF: p. 643 MSC: Client Needs: Health Promotion and Maintenance

13. A 50-year-old woman calls the clinic because she has noticed some changes in her body and breasts and wonders if these changes could be attributable to the hormone replacement therapy (HRT) she started 3 months earlier. The nurse should tell her: a. "HRT is at such a low dose that side effects are very unusual." b. "HRT has several side effects, including fluid retention, breast tenderness, and vaginal bleeding." c. "Vaginal bleeding with HRT is very unusual; I suggest you come into the clinic immediately to have this evaluated." d. "It sounds as if your dose of estrogen is too high; I think you may need to decrease the amount you are taking and then call back in a week."

ANS: B Side effects of HRT include fluid retention, breast pain, and vaginal bleeding. The other responses are not correct. DIF: Cognitive Level: Applying (Application) REF: p. 741 MSC: Client Needs: Physiologic Integrity: Pharmacologic and Parenteral Therapies

28. The nurse places a key in the hand of a patient and he identifies it as a penny. What term would the nurse use to describe this finding? a. Extinction b. Astereognosis c. Graphesthesia d. Tactile discrimination

ANS: B Stereognosis is the person's ability to recognize objects by feeling their forms, sizes, and weights. Astereognosis is an inability to identify objects correctly, and it occurs in sensory cortex lesions. Tactile discrimination tests fine touch. Extinction tests the person's ability to feel sensations on both sides of the body at the same point. DIF: Cognitive Level: Applying (Application) REF: p. 654 MSC: Client Needs: Physiologic Integrity: Physiologic Adaptation

14. A 52-year-old patient states that when she sneezes or coughs she "wets herself a little." She is very concerned that something may be wrong with her. The nurse suspects that the problem is: a. Dysuria. b. Stress incontinence. c. Hematuria. d. Urge incontinence.

ANS: B Stress incontinence is involuntary urine loss with physical strain, sneezing, or coughing. Dysuria is pain or burning with urination. Hematuria is bleeding with urination. Urge incontinence is involuntary urine loss that occurs as a result of an overactive detrusor muscle in the bladder that contracts and causes an urgent need to void. DIF: Cognitive Level: Understanding (Comprehension) REF: p. 742 MSC: Client Needs: Physiologic Integrity: Physiologic Adaptation

3. The nurse is performing an examination of the anus and rectum. Which of these statements is correct and important to remember during this examination? a. The rectum is approximately 8 cm long. b. The anorectal junction cannot be palpated. c. Above the anal canal, the rectum turns anteriorly. d. No sensory nerves are in the anal canal or rectum.

ANS: B The anal columns are folds of mucosa that extend vertically down from the rectum and end in the anorectal junction. This junction is not palpable but is visible on proctoscopy. The rectum is 12 cm long; just above the anal canal, the rectum dilates and turns posteriorly. DIF: Cognitive Level: Remembering (Knowledge) REF: p. 721 MSC: Client Needs: Safe and Effective Care Environment: Management of Care

13. During an interview the patient states, "I can feel this bump on the top of both of my shoulders—it doesn't hurt but I am curious about what it might be." The nurse should tell the patient that it is his: a. Subacromial bursa. b. Acromion process. c. Glenohumeral joint. d. Greater tubercle of the humerus.

ANS: B The bump of the scapula's acromion process is felt at the very top of the shoulder. The other options are not correct. DIF: Cognitive Level: Applying (Application) REF: p. 581 MSC: Client Needs: Physiologic Integrity: Physiologic Adaptation

44. During an examination, the nurse would expect the cervical os of a woman who has never had children to appear: a. Stellate. b. Small and round. c. As a horizontal irregular slit. d. Everted.

ANS: B The cervical os in a nulliparous woman is small and round. In the parous woman, it is a horizontal, irregular slit that also may show healed lacerations on the sides (see Figure 26-13). DIF: Cognitive Level: Understanding (Comprehension) REF: p. 750 MSC: Client Needs: Health Promotion and Maintenance

46. During an internal examination, the nurse notices that the cervix bulges outside the introitus when the patient is asked to strain. The nurse will document this as: a. Uterine prolapse, graded first degree. b. Uterine prolapse, graded second degree. c. Uterine prolapse, graded third degree. d. A normal finding.

ANS: B The cervix should not be found to bulge into the vagina. Uterine prolapse is graded as follows: first degree—the cervix appears at the introitus with straining; second degree—the cervix bulges outside the introitus with straining; and third degree—the whole uterus protrudes, even without straining (essentially, the uterus is inside out). DIF: Cognitive Level: Applying (Application) REF: p. 766 MSC: Client Needs: Physiologic Integrity: Physiologic Adaptation

22. When the nurse is discussing sexuality and sexual issues with an adolescent, a permission statement helps convey that it is normal to think or feel a certain way. Which statement is the best example of a permission statement? a. "It is okay that you have become sexually active." b. "Girls your age often have questions about sexual activity. Do you have any questions?" c. "If it is okay with you, I'd like to ask you some questions about your sexual history." d. "Girls your age often engage in sexual activities. It is okay to tell me if you have had intercourse."

ANS: B The examiner should start with a permission statement such as, "Girls your age often experience ..." A permission statement conveys the idea that it is normal to think or feel a certain way, and implying that the topic is normal and unexceptional is important. DIF: Cognitive Level: Understanding (Comprehension) REF: p. 743 MSC: Client Needs: Psychosocial Integrity

2. Which statement concerning the sphincters is correct? a. The internal sphincter is under voluntary control. b. The external sphincter is under voluntary control. c. Both sphincters remain slightly relaxed at all times. d. The internal sphincter surrounds the external sphincter.

ANS: B The external sphincter surrounds the internal sphincter but also has a small section overriding the tip of the internal sphincter at the opening. The external sphincter is under voluntary control. Except for the passing of feces and gas, the sphincters keep the anal canal tightly closed. DIF: Cognitive Level: Remembering (Knowledge) REF: p. 721 MSC: Client Needs: General

20. During an assessment of the CNs, the nurse finds the following: asymmetry when the patient smiles or frowns, uneven lifting of the eyebrows, sagging of the lower eyelids, and escape of air when the nurse presses against the right puffed cheek. This would indicate dysfunction of which of these CNs? a. Motor component of CN IV b. Motor component of CN VII c. Motor and sensory components of CN XI d. Motor component of CN X and sensory component of CN VII

ANS: B The findings listed reflect a dysfunction of the motor component of the facial nerve (CN VII). DIF: Cognitive Level: Analyzing (Analysis) REF: p. 646 MSC: Client Needs: Health Promotion and Maintenance

5. The nurse is preparing to conduct a mental status examination. Which statement is true regarding the mental status examination? a. A patient's family is the best resource for information about the patient's coping skills. b. Gathering mental status information during the health history interview is usually sufficient. c. Integrating the mental status examination into the health history interview takes an enormous amount of extra time. d. To get a good idea of the patient's level of functioning, performing a complete mental status examination is usually necessary.

ANS: B The full mental status examination is a systematic check of emotional and cognitive functioning. The steps described, however, rarely need to be taken in their entirety. Usually, one can assess mental status through the context of the health history interview. PTS: 1 DIF: Cognitive Level: Applying (Application) REF: p. 68 MSC: Client Needs: Psychosocial Integrity

17. The nurse is examining the hip area of a patient and palpates a flat depression on the upper, lateral side of the thigh when the patient is standing. The nurse interprets this finding as the: a. Ischial tuberosity. b. Greater trochanter. c. Iliac crest. d. Gluteus maximus muscle.

ANS: B The greater trochanter of the femur is palpated when the person is standing, and it appears as a flat depression on the upper lateral side of the thigh. The iliac crest is the upper part of the hip bone; the ischial tuberosity lies under the gluteus maximus muscle and is palpable when the hip is flexed; and the gluteus muscle is part of the buttocks. DIF: Cognitive Level: Understanding (Comprehension) REF: p. 582 MSC: Client Needs: Physiologic Integrity: Physiologic Adaptation

3. Which statement concerning the areas of the brain is true? a. The cerebellum is the center for speech and emotions. b. The hypothalamus controls body temperature and regulates sleep. c. The basal ganglia are responsible for controlling voluntary movements. d. Motor pathways of the spinal cord and brainstem synapse in the thalamus.

ANS: B The hypothalamus is a vital area with many important functions: body temperature controller, sleep center, anterior and posterior pituitary gland regulator, and coordinator of autonomic nervous system activity and emotional status. The cerebellum controls motor coordination, equilibrium, and balance. The basal ganglia control autonomic movements of the body. The motor pathways of the spinal cord synapse in various areas of the spinal cord, not in the thalamus.

12. A mother of a 1-month-old infant asks the nurse why it takes so long for infants to learn to roll over. The nurse knows that the reason for this is: a. A demyelinating process must be occurring with her infant. b. Myelin is needed to conduct the impulses, and the neurons of a newborn are not yet myelinated. c. The cerebral cortex is not fully developed; therefore, control over motor function gradually occurs. d. The spinal cord is controlling the movement because the cerebellum is not yet fully developed.

ANS: B The infant's sensory and motor development proceeds along with the gradual acquisition of myelin, which is needed to conduct most impulses. Very little cortical control exists, and the neurons are not yet myelinated. The other responses are not correct. DIF: Cognitive Level: Applying (Application) REF: p. 640 MSC: Client Needs: Health Promotion and Maintenance

16. The nurse is assessing a patient's ischial tuberosity. To palpate the ischial tuberosity, the nurse knows that it is best to have the patient: a. Standing. b. Flexing the hip. c. Flexing the knee. d. Lying in the supine position.

ANS: B The ischial tuberosity lies under the gluteus maximus muscle and is palpable when the hip is flexed. The other options are not correct. DIF: Cognitive Level: Understanding (Comprehension) REF: p. 582 MSC: Client Needs: Safe and Effective Care Environment: Management of Care

1. The two parts of the nervous system are the: a. Motor and sensory. b. Central and peripheral. c. Peripheral and autonomic. d. Hypothalamus and cerebral.

ANS: B The nervous system can be divided into two parts—central and peripheral. The central nervous system includes the brain and spinal cord. The peripheral nervous system includes the 12 pairs of cranial nerves (CNs), the 31 pairs of spinal nerves, and all of their branches.

24. During the examination portion of a patient's visit, she will be in lithotomy position. Which statement reflects some things that the nurse can do to make this position more comfortable for her? a. Ask her to place her hands and arms over her head. b. Elevate her head and shoulders to maintain eye contact. c. Allow her to choose to have her feet in the stirrups or have them resting side by side on the edge of the table. d. Allow her to keep her buttocks approximately 6 inches from the edge of the table to prevent her from feeling as if she will fall off.

ANS: B The nurse should elevate her head and shoulders to maintain eye contact. The patient's arms should be placed at her sides or across the chest. Placing her hands and arms over her head only tightens the abdominal muscles. The feet should be placed into the stirrups, knees apart, and buttocks at the edge of the examining table. The stirrups are placed so that the legs are not abducted too far. DIF: Cognitive Level: Applying (Application) REF: p. 745 MSC: Client Needs: Safe and Effective Care Environment: Management of Care

22. During the neurologic assessment of a "healthy" 35-year-old patient, the nurse asks him to relax his muscles completely. The nurse then moves each extremity through full range of motion. Which of these results would the nurse expect to find? a. Firm, rigid resistance to movement b. Mild, even resistance to movement c. Hypotonic muscles as a result of total relaxation d. Slight pain with some directions of movement

ANS: B Tone is the normal degree of tension (contraction) in voluntarily relaxed muscles. It shows a mild resistance to passive stretching. Normally, the nurse will notice a mild, even resistance to movement. The other responses are not correct. DIF: Cognitive Level: Applying (Application) REF: p. 647 MSC: Client Needs: Health Promotion and Maintenance

30. A patient calls the clinic for instructions before having a Papanicolaou (Pap) smear. The most appropriate instructions from the nurse are: a. "If you are menstruating, please use pads to avoid placing anything into the vagina." b. "Avoid intercourse, inserting anything into the vagina, or douching within 24 hours of your appointment." c. "If you suspect that you have a vaginal infection, please gather a sample of the discharge to bring with you." d. "We would like you to use a mild saline douche before your examination. You may pick this up in our office."

ANS: B When instructing a patient before Pap smear is obtained, the nurse should follow these guidelines: Do not obtain during the woman's menses or if a heavy infectious discharge is present. Instruct the woman not to douche, have intercourse, or put anything into the vagina within 24 hours before collecting the specimens. Any specimens will be obtained during the visit, not beforehand.

The nurse is assessing the joints of a woman who has stated, "I have a long family history of arthritis, and my joints hurt." The nurse suspects that she has osteoarthritis. Which of these are symptoms of osteoarthritis? Select all that apply. a. Symmetric joint involvement b. Asymmetric joint involvement c. Pain with motion of affected joints d. Affected joints are swollen with hard, bony protuberances e. Affected joints may have heat, redness, and swelling

ANS: B, C, D In osteoarthritis, asymmetric joint involvement commonly affects hands, knees, hips, and lumbar and cervical segments of the spine. Affected joints have stiffness, swelling with hard bony protuberances, pain with motion, and limitation of motion. The other options reflect the signs of rheumatoid arthritis. DIF: Cognitive Level: Applying (Application) REF: p. 620 MSC: Client Needs: Physiologic Integrity: Physiologic Adaptation

1. A 69-year-old patient has been admitted to an adult psychiatric unit because his wife thinks he is getting more and more confused. He laughs when he is found to be forgetful, saying "I'm just getting old!" After the nurse completes a thorough neurologic assessment, which findings would be indicative of Alzheimer disease? Select all that apply. a. Occasionally forgetting names or appointments b. Difficulty performing familiar tasks, such as placing a telephone call c. Misplacing items, such as putting dish soap in the refrigerator d. Sometimes having trouble finding the right word e. Rapid mood swings, from calm to tears, for no apparent reason f. Getting lost in one's own neighborhood

ANS: B, C, E, F Difficulty performing familiar tasks, misplacing items, rapid mood swings, and getting lost in one's own neighborhood can be warning signs of Alzheimer disease. Occasionally forgetting names or appointments, and sometimes having trouble finding the right word are part of normal aging. (For other examples of Alzheimer disease, see Table 23-2.) DIF: Cognitive Level: Applying (Application) REF: p. 677 MSC: Client Needs: Physiologic Integrity: Physiologic Adaptation

1. The nurse is palpating an ovarian mass during an internal examination of a 63-year-old woman. Which findings of the mass's characteristics would suggest the presence of an ovarian cyst? Select all that apply. a. Heavy and solid b. Mobile and fluctuant c. Mobile and solid d. Fixed e. Smooth and round f. Poorly defined

ANS: B, E An ovarian cyst (fluctuant ovarian mass) is usually asymptomatic and would feel like a smooth, round, fluctuant, mobile, nontender mass on the ovary. A mass that is heavy, solid, fixed, and poorly defined suggests malignancy. A benign mass may feel mobile and solid. DIF: Cognitive Level: Applying (Application) REF: p. 722 MSC: Client Needs: Physiologic Integrity: Physiologic Adaptation

10. A patient has a severed spinal nerve as a result of trauma. Which statement is true in this situation? a. Because there are 31 pairs of spinal nerves, no effect results if only one nerve is severed. b. The dermatome served by this nerve will no longer experience any sensation. c. The adjacent spinal nerves will continue to carry sensations for the dermatome served by the severed nerve. d. A severed spinal nerve will only affect motor function of the patient because spinal nerves have no sensory component.

ANS: C A dermatome is a circumscribed skin area that is primarily supplied from one spinal cord segment through a particular spinal nerve. The dermatomes overlap, which is a form of biologic insurance; that is, if one nerve is severed, then most of the sensations can be transmitted by the spinal nerve above and the spinal nerve below the severed nerve. DIF: Cognitive Level: Applying (Application) REF: p. 639 MSC: Client Needs: Physiologic Integrity: Physiologic Adaptation

24. A teenage girl has arrived complaining of pain in her left wrist. She was playing basketball when she fell and landed on her left hand. The nurse examines her hand and would expect a fracture if the girl complains of a: a. Dull ache. b. Deep pain in her wrist. c. Sharp pain that increases with movement. d. Dull throbbing pain that increases with rest.

ANS: C A fracture causes sharp pain that increases with movement. The other types of pain do not occur with a fracture. DIF: Cognitive Level: Analyzing (Analysis) REF: p. 586 MSC: Client Needs: Physiologic Integrity: Physiologic Adaptation

31. The nurse is planning health teaching for a 65-year-old woman who has had a cerebrovascular accident (stroke) and has aphasia. Which of these questions is most important to use when assessing mental status in this patient? a. "Please count backward from 100 by seven." b. "I will name three items and ask you to repeat them in a few minutes." c. "Please point to articles in the room and parts of the body as I name them." d. "What would you do if you found a stamped, addressed envelope on the sidewalk?"

ANS: C Additional tests for persons with aphasia include word comprehension (asking the individual to point to articles in the room or parts of the body), reading (asking the person to read available print), and writing (asking the person to make up and write a sentence). PTS: 1 DIF: Cognitive Level: Applying (Application) REF: p. 71 MSC: Client Needs: Psychosocial Integrity

45. The nurse is assessing a 1-week-old infant and is testing his muscle strength. The nurse lifts the infant with hands under the axillae and notices that the infant starts to "slip" between the hands. The nurse should: a. Suspect a fractured clavicle. b. Suspect that the infant may have a deformity of the spine. c. Suspect that the infant may have weakness of the shoulder muscles. d. Conclude that this is a normal finding because the musculature of an infant at this age is undeveloped.

ANS: C An infant who starts to "slip" between the nurse's hands shows weakness of the shoulder muscles. An infant with normal muscle strength wedges securely between the nurse's hands. The other responses are not correct. DIF: Cognitive Level: Analyzing (Analysis) REF: p. 612 MSC: Client Needs: Health Promotion and Maintenance

12. The nurse is planning to assess new memory with a patient. The best way for the nurse to do this would be to: a. Administer the FACT test. b. Ask him to describe his first job. c. Give him the Four Unrelated Words Test. d. Ask him to describe what television show he was watching before coming to the clinic.

ANS: C Ask questions that can be corroborated, which screens for the occasional person who confabulates or makes up answers to fill in the gaps of memory loss. The Four Unrelated Words Test tests the person's ability to lay down new memories and is a highly sensitive and valid memory test. PTS: 1 DIF: Cognitive Level: Applying (Application) REF: p. 71 MSC: Client Needs: Psychosocial Integrity

16. When taking the health history on a patient with a seizure disorder, the nurse assesses whether the patient has an aura. Which of these would be the best question for obtaining this information? a. "Does your muscle tone seem tense or limp?" b. "After the seizure, do you spend a lot of time sleeping?" c. "Do you have any warning sign before your seizure starts?" d. "Do you experience any color change or incontinence during the seizure?"

ANS: C Aura is a subjective sensation that precedes a seizure; it could be auditory, visual, or motor. The other questions do not solicit information about an aura. DIF: Cognitive Level: Applying (Application) REF: p. 641 MSC: Client Needs: Physiologic Integrity: Physiologic Adaptation

7. Generally, the changes normally associated with menopause occur because the cells in the reproductive tract are: a. Aging. b. Becoming fibrous. c. Estrogen dependent. d. Able to respond to estrogen.

ANS: C Because cells in the reproductive tract are estrogen dependent, decreased estrogen levels during menopause bring dramatic physical changes. The other options are not correct. DIF: Cognitive Level: Remembering (Knowledge) REF: p. 740 MSC: Client Needs: Health Promotion and Maintenance

10. A patient who is visiting the clinic complains of having "stomach pains for 2 weeks" and describes his stools as being "soft and black" for approximately the last 10 days. He denies taking any medications. The nurse is aware that these symptoms are mostly indicative of: a. Excessive fat caused by malabsorption. b. Increased iron intake, resulting from a change in diet. c. Occult blood, resulting from gastrointestinal bleeding. d. Absent bile pigment from liver problems.

ANS: C Black stools may be tarry as a result of occult blood (melena) from gastrointestinal bleeding or nontarry from ingestion of iron medications (not diet). Excessive fat causes the stool to become frothy. The absence of bile pigment causes clay-colored stools. DIF: Cognitive Level: Analyzing (Analysis) REF: p. 724 MSC: Client Needs: Physiologic Integrity: Physiologic Adaptation

40. A patient who has had rheumatoid arthritis for years comes to the clinic to ask about changes in her fingers. The nurse will assess for signs of what problems? a. Heberden nodes b. Bouchard nodules c. Swan-neck deformities d. Dupuytren contractures

ANS: C Changes in the fingers caused by chronic rheumatoid arthritis include swan-neck and boutonniere deformities. Heberden nodes and Bouchard nodules are associated with osteoarthritis. Dupuytren contractures of the digits occur because of chronic hyperplasia of the palmar fascia (see Table 22-4). DIF: Cognitive Level: Applying (Application) REF: p. 624 MSC: Client Needs: Physiologic Integrity: Physiologic Adaptation

21. A patient repeatedly seems to have difficulty coming up with a word. He says, "I was on my way to work, and when I got there, the thing that you step into that goes up in the air was so full that I decided to take the stairs." The nurse will note on his chart that he is using or experiencing: a. Blocking b. Neologism c. Circumlocution d. Circumstantiality

ANS: C Circumlocution is a roundabout expression, substituting a phrase when one cannot think of the name of the object. PTS: 1 DIF: Cognitive Level: Understanding (Comprehension) REF: p. 84 MSC: Client Needs: Psychosocial Integrity

53. A man who was found wandering in a park at 2 AM has been brought to the emergency department for an examination; he said he fell and hit his head. During the examination, the nurse asks him to use his index finger to touch the nurse's finger, then his own nose, then the nurse's finger again (which has been moved to a different location). The patient is clumsy, unable to follow the instructions, and overshoots the mark, missing the finger. The nurse should suspect which of the following? a. Cerebral injury b. Cerebrovascular accident c. Acute alcohol intoxication d. Peripheral neuropathy

ANS: C During the finger-to-finger test, if the person has clumsy movement with overshooting the mark, either a cerebellar disorder or acute alcohol intoxication should be suspected. The person's movements should be smooth and accurate. The other options are not correct. DIF: Cognitive Level: Analyzing (Analysis) REF: p. 648 MSC: Client Needs: Physiologic Integrity: Physiologic Adaptation

22. During an examination, the nurse notes that a patient is exhibiting flight of ideas. Which statement by the patient is an example of flight of ideas? a. "My stomach hurts. Hurts, spurts, burts." b. "Kiss, wood, reading, ducks, onto, maybe." c. "Take this pill? The pill is red. I see red. Red velvet is soft, soft as a baby's bottom." d. "I wash my hands, wash them, wash them. I usually go to the sink and wash my hands."

ANS: C Flight of ideas is demonstrated by an abrupt change, rapid skipping from topic to topic, and practically continuous flow of accelerated speech. Topics usually have recognizable associations or are plays on words. PTS: 1 DIF: Cognitive Level: Understanding (Comprehension) REF: p. 84 MSC: Client Needs: Psychosocial Integrity

35. A mother brings her newborn baby boy in for a checkup; she tells the nurse that he does not seem to be moving his right arm as much as his left and that he seems to have pain when she lifts him up under the arms. The nurse suspects a fractured clavicle and would observe for: a. Negative Allis test. b. Positive Ortolani sign. c. Limited range of motion during the Moro reflex. d. Limited range of motion during Lasègue test.

ANS: C For a fractured clavicle, the nurse should observe for limited arm range of motion and unilateral response to the Moro reflex. The other tests are not appropriate for this type of fracture. DIF: Cognitive Level: Analyzing (Analysis) REF: p. 611 MSC: Client Needs: Health Promotion and Maintenance

29. The nurse suspects that a patient has carpal tunnel syndrome and wants to perform the Phalen test. To perform this test, the nurse should instruct the patient to: a. Dorsiflex the foot. b. Plantarflex the foot. c. Hold both hands back to back while flexing the wrists 90 degrees for 60 seconds. d. Hyperextend the wrists with the palmar surface of both hands touching, and wait for 60 seconds.

ANS: C For the Phalen test, the nurse should ask the person to hold both hands back to back while flexing the wrists 90 degrees. Acute flexion of the wrist for 60 seconds produces no symptoms in the normal hand. The Phalen test reproduces numbness and burning in a person with carpal tunnel syndrome. The other actions are not correct when testing for carpal tunnel syndrome. DIF: Cognitive Level: Applying (Application) REF: p. 599 MSC: Client Needs: Physiologic Integrity: Physiologic Adaptation

11. A patient has had three pregnancies and two live births. The nurse would record this information as grav _____, para _____, AB _____. a. 2; 2; 1 b. 3; 2; 0 c. 3; 2; 1 d. 3; 3; 1

ANS: C Gravida (grav) is the number of pregnancies. Para is the number of births. Abortions are interrupted pregnancies, including elective abortions and spontaneous miscarriages. DIF: Cognitive Level: Applying (Application) REF: p. 741 MSC: Client Needs: Safe and Effective Care Environment: Management of Care

43. During an external genitalia examination of a woman, the nurse notices several lesions around the vulva. The lesions are pink, moist, soft, and pointed papules. The patient states that she is not aware of any problems in that area. The nurse recognizes that these lesions may be: a. Syphilitic chancre. b. Herpes simplex virus type 2 (herpes genitalis). c. HPV or genital warts. d. Pediculosis pubis (crab lice).

ANS: C HPV lesions are painless, warty growths that the woman may not notice. Lesions are pink or flesh colored, soft, pointed, moist, warty papules that occur in single or multiple cauliflower-like patches around the vulva, introitus, anus, vagina, or cervix. Herpetic lesions are painful clusters of small, shallow vesicles with surrounding erythema. Syphilitic chancres begin as a solitary silvery papule that erodes into a red, round or oval superficial ulcer with a yellowish discharge. Pediculosis pubis causes severe perineal itching and excoriations and erythematous areas (see Table 26-2). DIF: Cognitive Level: Analyzing (Analysis) REF: p. 765 MSC: Client Needs: Physiologic Integrity: Physiologic Adaptation

12. During the interview with a female patient, the nurse gathers data that indicate the patient is perimenopausal. Which of these statements made by this patient leads to this conclusion? a. "I have noticed that my muscles ache at night when I go to bed." b. "I will be very happy when I can stop worrying about having a period." c. "I have been noticing that I sweat a lot more than I used to, especially at night." d. "I have only been pregnant twice, but both times I had breast tenderness as my first symptom."

ANS: C Hormone shifts occur during the perimenopausal period, and associated symptoms of menopause may occur, such as hot flashes, night sweats, numbness and tingling, headache, palpitations, drenching sweats, mood swings, vaginal dryness, and itching. The other responses are not correct. DIF: Cognitive Level: Analyzing (Analysis) REF: p. 741 MSC: Client Needs: Health Promotion and Maintenance

7. The nurse is conducting a patient interview. Which statement made by the patient should the nurse more fully explore during the interview? a. "I sleep like a baby." b. "I have no health problems." c. "I never did too good in school." d. "I am not currently taking any medications."

ANS: C In every mental status examination, the following factors from the health history that could affect the findings should be noted: any known illnesses or health problems, such as alcoholism or chronic renal disease; current medications, the side effects of which may cause confusion or depression; the usual educational and behavioral level, noting this level as the patient's normal baseline and not expecting a level of performance on the mental status examination to exceed it; and responses to personal history questions, indicating current stress, social interaction patterns, and sleep habits. PTS: 1 DIF: Cognitive Level: Analyzing (Analysis) REF: p. 69 MSC: Client Needs: Psychosocial Integrity

32. During an examination, the nurse asks a patient to bend forward from the waist and notices that the patient has lateral tilting. When his leg is raised straight up, the patient complains of a pain going down his buttock into his leg. The nurse suspects: a. Scoliosis. b. Meniscus tear. c. Herniated nucleus pulposus. d. Spasm of paravertebral muscles.

ANS: C Lateral tilting and sciatic pain with straight leg raising are findings that occur with a herniated nucleus pulposus. The other options are not correct. DIF: Cognitive Level: Analyzing (Analysis) REF: p. 606 |p. 609 MSC: Client Needs: Physiologic Integrity: Physiologic Adaptation

19. The nurse is explaining the mechanism of the growth of long bones to a mother of a toddler. Where does lengthening of the bones occur? a. Bursa b. Calcaneus c. Epiphyses d. Tuberosities

ANS: C Lengthening occurs at the epiphyses, or growth plates. The other options are not correct. DIF: Cognitive Level: Understanding (Comprehension) REF: p. 584 MSC: Client Needs: Health Promotion and Maintenance

27. The nurse is performing a neurologic assessment on a 41-year-old woman with a history of diabetes. When testing her ability to feel the vibrations of a tuning fork, the nurse notices that the patient is unable to feel vibrations on the great toe or ankle bilaterally, but she is able to feel vibrations on both patellae. Given this information, what would the nurse suspect? a. Hyperalgesia b. Hyperesthesia c. Peripheral neuropathy d. Lesion of sensory cortex

ANS: C Loss of vibration sense occurs with peripheral neuropathy (e.g., diabetes and alcoholism). Peripheral neuropathy is worse at the feet and gradually improves as the examiner moves up the leg, as opposed to a specific nerve lesion, which has a clear zone of deficit for its dermatome. The other responses are incorrect. DIF: Cognitive Level: Analyzing (Analysis) REF: p. 653 MSC: Client Needs: Physiologic Integrity: Physiologic Adaptation

27. A patient states, "I feel so sad all of the time. I can't feel happy even doing things I used to like to do." He also states that he is tired, sleeps poorly, and has no energy. To differentiate between a dysthymic disorder and a major depressive disorder, the nurse should ask which question? a. "Have you had any weight changes?" b. "Are you having any thoughts of suicide?" c. "How long have you been feeling this way?" d. "Are you having feelings of worthlessness?"

ANS: C Major depressive disorder is characterized by one or more major depressive episodes, that is, at least 2 weeks of depressed mood or loss of interest accompanied by at least four additional symptoms of depression. Dysthymic disorder is characterized by at least 2 years of depressed mood for more days than not, accompanied by additional depressive symptoms. PTS: 1 DIF: Cognitive Level: Analyzing (Analysis) REF: p. 73 MSC: Client Needs: Psychosocial Integrity

1. During an examination, the nurse can assess mental status by which activity? a. Examining the patient's electroencephalogram b. Observing the patient as he or she performs an intelligence quotient (IQ) test c. Observing the patient and inferring health or dysfunction d. Examining the patient's response to a specific set of questions

ANS: C Mental status cannot be directly scrutinized like the characteristics of skin or heart sounds. Its functioning is inferred through an assessment of an individual's behaviors, such as consciousness, language, mood and affect, and other aspects. PTS: 1 DIF: Cognitive Level: Understanding (Comprehension) REF: p. 67 MSC: Client Needs: Psychosocial Integrity

29. The nurse is performing a mental status examination. Which statement is true regarding the assessment of mental status? a. Mental status assessment diagnoses specific psychiatric disorders. b. Mental disorders occur in response to everyday life stressors. c. Mental status functioning is inferred through the assessment of an individual's behaviors. d. Mental status can be directly assessed, similar to other systems of the body (e.g., heart sounds, breath sounds).

ANS: C Mental status functioning is inferred through the assessment of an individual's behaviors. It cannot be directly assessed like the characteristics of the skin or heart sounds. PTS: 1 DIF: Cognitive Level: Understanding (Comprehension) REF: p. 67 MSC: Client Needs: Psychosocial Integrity

1. A patient is being assessed for range-of-joint movement. The nurse asks him to move his arm in toward the center of his body. This movement is called: a. Flexion. b. Abduction. c. Adduction. d. Extension.

ANS: C Moving a limb toward the midline of the body is called adduction; moving a limb away from the midline of the body is called abduction. Flexion is bending a limb at a joint; and extension is straightening a limb at a joint. DIF: Cognitive Level: Understanding (Comprehension) REF: p. 578 MSC: Client Needs: Physiologic Integrity: Physiologic Adaptation

29. The nurse is examining a 35-year-old female patient. During the health history, the nurse notices that she has had two term pregnancies, and both babies were delivered vaginally. During the internal examination, the nurse observes that the cervical os is a horizontal slit with some healed lacerations and that the cervix has some nabothian cysts that are small, smooth, and yellow. In addition, the nurse notices that the cervical surface is granular and red, especially around the os. Finally, the nurse notices the presence of stringy, opaque, odorless secretions. Which of these findings are abnormal? a. Nabothian cysts are present. b. The cervical os is a horizontal slit. c. The cervical surface is granular and red. d. Stringy and opaque secretions are present.

ANS: C Normal findings: Nabothian cysts may be present on the cervix after childbirth. The cervical os is a horizontal, irregular slit in the parous woman. Secretions vary according to the day of the menstrual cycle, and may be clear and thin or thick, opaque, and stringy. The surface is normally smooth, but cervical eversion, or ectropion, may occur where the endocervical canal is rolled out. Abnormal finding: The cervical surface should not be reddened or granular, which may indicate a lesion. DIF: Cognitive Level: Analyzing (Analysis) REF: p. 750 MSC: Client Needs: Health Promotion and Maintenance

When performing the bimanual examination, the nurse notices that the cervix feels smooth and firm, is round, and is fixed in place (does not move). When cervical palpation is performed, the patient complains of some pain. The nurse's interpretation of these results should be which of these? a. These findings are all within normal limits. b. Cervical consistency should be soft and velvety—not firm. c. The cervix should move when palpated; an immobile cervix may indicate malignancy. d. Pain may occur during palpation of the cervix.

ANS: C Normally, the cervix feels smooth and firm, similar to the consistency of the tip of the nose. It softens and feels velvety at 5 to 6 weeks of pregnancy (Goodell sign). The cervix should be evenly rounded. With a finger on either side, the examiner should be able to move the cervix gently from side to side, and doing so should produce no pain for the patient. Hardness of the cervix may occur with malignancy. Immobility may occur with malignancy, and pain may occur with inflammation or ectopic pregnancy.

21. When the nurse is interviewing a preadolescent girl, which opening question would be least threatening? a. "Do you have any questions about growing up?" b. "What has your mother told you about growing up?" c. "When did you notice that your body was changing?" d. "I remember being very scared when I got my period. How do you think you'll feel?"

ANS: C Open-ended questions such as, "When did you ...?" rather than "Do you ...?" should be asked. Open-ended questions are less threatening because they imply that the topic is normal and unexceptional. DIF: Cognitive Level: Understanding (Comprehension) REF: p. 743 MSC: Client Needs: Psychosocial Integrity

42. A 14-year-old boy who has been diagnosed with Osgood-Schlatter disease reports painful swelling just below the knee for the past 5 months. Which response by the nurse is appropriate? a. "If these symptoms persist, you may need arthroscopic surgery." b. "You are experiencing degeneration of your knee, which may not resolve." c. "Your disease is due to repeated stress on the patellar tendon. It is usually self-limited, and your symptoms should resolve with rest." d. "Increasing your activity and performing knee-strengthening exercises will help decrease the inflammation and maintain mobility in the knee."

ANS: C Osgood-Schlatter disease is a painful swelling of the tibial tubercle just below the knee and most likely due to repeated stress on the patellar tendon. It is usually self-limited, occurring during rapid growth and most often in boys. The symptoms resolve with rest. The other responses are not appropriate. DIF: Cognitive Level: Applying (Application) REF: p. 626 MSC: Client Needs: Physiologic Integrity: Physiologic Adaptation

41. During a bimanual examination, the nurse detects a solid tumor on the ovary that is heavy and fixed, with a poorly defined mass. This finding is suggestive of: a. Ovarian cyst. b. Endometriosis. c. Ovarian cancer. d. Ectopic pregnancy.

ANS: C Ovarian tumors that are solid, heavy, and fixed, with poorly defined mass are suggestive of malignancy. Benign masses may feel mobile and solid. An ovarian cyst may feel smooth, round, fluctuant, mobile, and nontender. With an ectopic pregnancy, the examiner may feel a palpable, tender pelvic mass that is solid, mobile, and unilateral. Endometriosis may have masses (in various locations in the pelvic area) that are small, firm, nodular, and tender to palpation, with enlarged ovaries. DIF: Cognitive Level: Applying (Application) REF: p. 772 MSC: Client Needs: Physiologic Integrity: Physiologic Adaptation

23. The nurse is teaching a class on preventing osteoporosis to a group of perimenopausal women. Which of these actions is the best way to prevent or delay bone loss in this group? a. Taking calcium and vitamin D supplements b. Taking medications to prevent osteoporosis c. Performing physical activity, such as fast walking d. Assessing bone density annually

ANS: C Physical activity, such as fast walking, delays or prevents bone loss in perimenopausal women. The faster the pace of walking, the higher the preventive effect is on the risk of hip fracture. The other options are not correct. DIF: Cognitive Level: Applying (Application) REF: p. 585 MSC: Client Needs: Health Promotion and Maintenance

34. During a neonatal examination, the nurse notices that the newborn infant has six toes. This finding is documented as: a. Unidactyly. b. Syndactyly. c. Polydactyly. d. Multidactyly.

ANS: C Polydactyly is the presence of extra fingers or toes. Syndactyly is webbing between adjacent fingers or toes. The other terms are not correct. DIF: Cognitive Level: Understanding (Comprehension) REF: p. 611 MSC: Client Needs: Physiologic Integrity: Physiologic Adaptation

15. During the interview, a patient reveals that she has some vaginal discharge. She is worried that it may be a sexually transmitted infection. The nurse's most appropriate response to this would be: a. "Oh, don't worry. Some cyclic vaginal discharge is normal." b. "Have you been engaging in unprotected sexual intercourse?" c. "I'd like some information about the discharge. What color is it?" d. "Have you had any urinary incontinence associated with the discharge?"

ANS: C Questions that help the patient reveal more information about her symptoms should be asked in a nonthreatening manner. Asking about the amount, color, and odor of the vaginal discharge provides the opportunity for further assessment. Normal vaginal discharge is small, clear or cloudy, and always nonirritating. DIF: Cognitive Level: Analyzing (Analysis) REF: p. 742 MSC: Client Needs: Physiologic Integrity: Physiologic Adaptation

25. A patient is complaining of pain in his joints that is worse in the morning, better after he moves around for a while, and then gets worse again if he sits for long periods. The nurse should assess for other signs of what problem? a. Tendinitis b. Osteoarthritis c. Rheumatoid arthritis d. Intermittent claudication

ANS: C Rheumatoid arthritis is worse in the morning when a person arises. Movement increases most joint pain, except the pain with rheumatoid arthritis, which decreases with movement. The other options are not correct.

16. A woman states that 2 weeks ago she had a urinary tract infection that was treated with an antibiotic. As a part of the interview, the nurse should ask, "Have you noticed any: a. "Changes in your urination patterns?" b. "Excessive vaginal bleeding?" c. "Unusual vaginal discharge or itching?" d. "Changes in your desire for intercourse?"

ANS: C Several medications may increase the risk of vaginitis. Broad-spectrum antibiotics alter the balance of normal flora, which may lead to the development of vaginitis. The other questions are not appropriate. DIF: Cognitive Level: Applying (Application) REF: p. 742 MSC: Client Needs: Physiologic Integrity: Pharmacologic and Parenteral Therapies

48. A patient is unable to perform rapid alternating movements such as rapidly patting her knees. The nurse should document this inability as: a. Ataxia. b. Astereognosis. c. Presence of dysdiadochokinesia. d. Loss of kinesthesia.

ANS: C Slow clumsy movements and the inability to perform rapid alternating movements occur with cerebellar disease. The condition is termed dysdiadochokinesia. Ataxia is an uncoordinated or unsteady gait. Astereognosis is the inability to identify an object by feeling it. Kinesthesia is the person's ability to perceive passive movement of the extremities or the loss of position sense. DIF: Cognitive Level: Applying (Application) REF: p. 648 MSC: Client Needs: Physiologic Integrity: Physiologic Adaptation

13. During an assessment of an 80-year-old patient, the nurse notices the following: an inability to identify vibrations at her ankle and to identify the position of her big toe, a slower and more deliberate gait, and a slightly impaired tactile sensation. All other neurologic findings are normal. The nurse should interpret that these findings indicate: a. CN dysfunction. b. Lesion in the cerebral cortex. c. Normal changes attributable to aging. d. Demyelination of nerves attributable to a lesion.

ANS: C Some aging adults show a slower response to requests, especially for those calling for coordination of movements. The findings listed are normal in the absence of other significant abnormal findings. The other responses are incorrect. DIF: Cognitive Level: Analyzing (Analysis) REF: p. 640 MSC: Client Needs: Health Promotion and Maintenance

40. The nurse is caring for a patient who has just had neurosurgery. To assess for increased intracranial pressure, what would the nurse include in the assessment? a. CNs, motor function, and sensory function b. Deep tendon reflexes, vital signs, and coordinated movements c. Level of consciousness, motor function, pupillary response, and vital signs d. Mental status, deep tendon reflexes, sensory function, and pupillary response

ANS: C Some hospitalized persons have head trauma or a neurologic deficit from a systemic disease process. These people must be closely monitored for any improvement or deterioration in neurologic status and for any indication of increasing intracranial pressure. The nurse should use an abbreviation of the neurologic examination in the following sequence: level of consciousness, motor function, pupillary response, and vital signs. DIF: Cognitive Level: Applying (Application) REF: p. 670 MSC: Client Needs: Physiologic Integrity: Physiologic Adaptation

18. The nurse is performing the Denver II screening test on a 12-month-old infant during a routine well-child visit. The nurse should tell the infant's parents that the Denver II: a. Tests three areas of development: cognitive, physical, and psychological b. Will indicate whether the child has a speech disorder so that treatment can begin. c. Is a screening instrument designed to detect children who are slow in development. d. Is a test to determine intellectual ability and may indicate whether problems will develop later in school.

ANS: C The Denver II is a screening instrument designed to detect developmental delays in infants and preschoolers. It tests four functions: gross motor, language, fine motor-adaptive, and personal-social. The Denver II is not an intelligence test; it does not predict current or future intellectual ability. It is not diagnostic; it does not suggest treatment regimens. PTS: 1 DIF: Cognitive Level: Applying (Application) REF: p. 75 MSC: Client Needs: Psychosocial Integrity

13. A 45-year-old woman is at the clinic for a mental status assessment. In giving her the Four Unrelated Words Test, the nurse would be concerned if she could not ____ four unrelated words ____. a. Invent; within 5 minutes b. Invent; within 30 seconds c. Recall; after a 30-minute delay d. Recall; after a 60-minute delay

ANS: C The Four Unrelated Words Test tests the person's ability to lay down new memories. It is a highly sensitive and valid memory test. It requires more effort than the recall of personal or historic events. To the person say, "I am going to say four words. I want you to remember them. In a few minutes I will ask you to recall them." After 5 minutes, ask for the four words. The normal response for persons under 60 years is an accurate three- or four-word recall after a 5-, 10-, and 30-minute delay. PTS: 1 DIF: Cognitive Level: Analyzing (Analysis) REF: p. 71 MSC: Client Needs: Psychosocial Integrity

33. The nurse is providing instructions to newly hired graduates for the mini-mental state examination (MMSE). Which statement best describes this examination? a. Scores below 30 indicate cognitive impairment. b. The MMSE is a good tool to evaluate mood and thought processes. c. This examination is a good tool to detect delirium and dementia and to differentiate these from psychiatric mental illness. d. The MMSE is useful tool for an initial evaluation of mental status. Additional tools are needed to evaluate cognition changes over time.

ANS: C The MMSE is a quick, easy test of 11 questions and is used for initial and serial evaluations and can demonstrate a worsening or an improvement of cognition over time and with treatment. It evaluates cognitive functioning, not mood or thought processes. MMSE is a good screening tool to detect dementia and delirium and to differentiate these from psychiatric mental illness. PTS: 1 DIF: Cognitive Level: Understanding (Comprehension) REF: p. 75 MSC: Client Needs: Psychosocial Integrity

36. While assessing a 7-month-old infant, the nurse makes a loud noise and notices the following response: abduction and flexion of the arms and legs; fanning of the fingers, and curling of the index finger and thumb in a C position, followed by the infant bringing in the arms and legs to the body. What does the nurse know about this response? a. This response could indicate brachial nerve palsy. b. This reaction is an expected startle response at this age. c. This reflex should have disappeared between 1 and 4 months of age. d. This response is normal as long as the movements are bilaterally symmetric.

ANS: C The Moro reflex is present at birth and usually disappears at 1 to 4 months. Absence of the Moro reflex in the newborn or its persistence after 5 months of age indicates severe central nervous system injury. The other responses are incorrect. DIF: Cognitive Level: Analyzing (Analysis) REF: p. 666 MSC: Client Needs: Health Promotion and Maintenance

3. The nurse is assessing a 75-year-old man. As the nurse begins the mental status portion of the assessment, the nurse expects that this patient: a. Will have no decrease in any of his abilities, including response time. b. Will have difficulty on tests of remote memory because this ability typically decreases with age. c. May take a little longer to respond, but his general knowledge and abilities should not have declined. d. Will exhibit had a decrease in his response time because of the loss of language and a decrease in general knowledge.

ANS: C The aging process leaves the parameters of mental status mostly intact. General knowledge does not decrease, and little or no loss in vocabulary occurs. Response time is slower than in a youth. It takes a little longer for the brain to process information and to react to it. Recent memory, which requires some processing, is somewhat decreased with aging, but remote memory is not affected. PTS: 1 DIF: Cognitive Level: Analyzing (Analysis) REF: p. 68 MSC: Client Needs: Psychosocial Integrity

7. The articulation of the mandible and the temporal bone is known as the: a. Intervertebral foramen. b. Condyle of the mandible. c. Temporomandibular joint. d. Zygomatic arch of the temporal bone.

ANS: C The articulation of the mandible and the temporal bone is the temporomandibular joint. The other responses are not correct. DIF: Cognitive Level: Remembering (Knowledge) REF: p. 578 MSC: Client Needs: General

8. A 30-year-old woman tells the nurse that she has been very unsteady and has had difficulty in maintaining her balance. Which area of the brain that is related to these findings would concern the nurse? a. Thalamus b. Brainstem c. Cerebellum d. Extrapyramidal tract

ANS: C The cerebellar system coordinates movement, maintains equilibrium, and helps maintain posture. The thalamus is the primary relay station where sensory pathways of the spinal cord, cerebellum, and brainstem form synapses on their way to the cerebral cortex. The brainstem consists of the midbrain, pons, and medulla and has various functions, especially concerning autonomic centers. The extrapyramidal tract maintains muscle tone for gross automatic movements, such as walking. DIF: Cognitive Level: Understanding (Comprehension) REF: p. 637 MSC: Client Needs: Physiologic Integrity: Physiologic Adaptation

27. The nurse is preparing for an internal genitalia examination of a woman. Which order of the examination is correct? a. Bimanual, speculum, and rectovaginal b. Speculum, rectovaginal, and bimanual c. Speculum, bimanual, and rectovaginal d. Rectovaginal, bimanual, and speculum

ANS: C The correct sequence is speculum examination, then bimanual examination after removing the speculum, and then rectovaginal examination. The examiner should change gloves before performing the rectovaginal examination to avoid spreading any possible infection. DIF: Cognitive Level: Analyzing (Analysis) REF: p. 748 |p. 754 |p. 758 MSC: Client Needs: Safe and Effective Care Environment: Management of Care

28. During an internal examination of a woman's genitalia, the nurse will use which technique for proper insertion of the speculum? a. The woman is instructed to bear down, the speculum blades are opened and applied in a swift, upward movement. b. The blades of the speculum are inserted on a horizontal plane, turning them to a 30-degree angle while continuing to insert them. The woman is asked to bear down after the speculum is inserted. c. The woman is instructed to bear down, the width of the blades are horizontally turned, and the speculum is inserted downward at a 45-degree angle toward the small of the woman's back. d. The blades are locked open by turning the thumbscrew. Once the blades are open, pressure is applied to the introitus and the blades are inserted downward at a 45-degree angle to bring the cervix into view.

ANS: C The examiner should instruct the woman to bear down, turn the width of the blades horizontally, and insert the speculum at a 45-degree angle downward toward the small of the woman's back. (See the text under "Speculum Examination" for more detail.) DIF: Cognitive Level: Applying (Application) REF: p. 749 MSC: Client Needs: Safe and Effective Care Environment: Management of Care

15. A patient is visiting the clinic for an evaluation of a swollen, painful knuckle. The nurse notices that the knuckle above his ring on the left hand is swollen and that he is unable to remove his wedding ring. This joint is called the _________ joint. a. Interphalangeal b. Tarsometatarsal c. Metacarpophalangeal d. Tibiotalar

ANS: C The joint located just above the ring on the finger is the metacarpophalangeal joint. The interphalangeal joint is located distal to the metacarpophalangeal joint. The tarsometatarsal and tibiotalar joints are found in the foot and ankle. (See Figure 22-10 for a diagram of the bones and joints of the hand and fingers.) DIF: Cognitive Level: Understanding (Comprehension) REF: p. 582 MSC: Client Needs: Physiologic Integrity: Physiologic Adaptation

31. When the nurse is testing the triceps reflex, what is the expected response? a. Flexion of the hand b. Pronation of the hand c. Extension of the forearm d. Flexion of the forearm

ANS: C The normal response of the triceps reflex is extension of the forearm. The normal response of the biceps reflex causes flexion of the forearm. The other responses are incorrect. DIF: Cognitive Level: Remembering (Knowledge) REF: p. 657 MSC: Client Needs: Safe and Effective Care Environment: Management of Care

10. A patient has been in the intensive care unit for 10 days. He has just been moved to the medical-surgical unit, and the admitting nurse is planning to perform a mental status examination. During the tests of cognitive function, the nurse would expect that he: a. May display some disruption in thought content. b. Will state, "I am so relieved to be out of intensive care." c. Will be oriented to place and person, but the patient may not be certain of the date. d. May show evidence of some clouding of his level of consciousness.

ANS: C The nurse can discern the orientation of cognitive function through the course of the interview or can directly and tactfully ask, "Some people have trouble keeping up with the dates while in the hospital. Do you know today's date?" Many hospitalized people have trouble with the exact date but are fully oriented on the remaining items. PTS: 1 DIF: Cognitive Level: Analyzing (Analysis) REF: pp. 70-71 MSC: Client Needs: Psychosocial Integrity

17. Which statement would be most appropriate when the nurse is introducing the topic of sexual relationships during an interview? a. "Now, it is time to talk about your sexual history. When did you first have intercourse?" b. "Women often feel dissatisfied with their sexual relationships. Would it be okay to discuss this now?" c. "Women often have questions about their sexual relationship and how it affects their health. Do you have any questions?" d. "Most women your age have had more than one sexual partner. How many would you say you have had?"

ANS: C The nurse should begin with an open-ended question to assess individual needs. The nurse should include appropriate questions as a routine part of the health history, because doing so communicates that the nurse accepts the individual's sexual activity and believes it is important. The nurse's comfort with the discussion prompts the patient's interest and, possibly, relief that the topic has been introduced. The initial discussion establishes a database for comparison with any future sexual activities and provides an opportunity to screen sexual problems. DIF: Cognitive Level: Applying (Application) REF: p. 742 MSC: Client Needs: Psychosocial Integrity

13. The nurse is examining only the rectal area of a woman and should place the woman in what position? a. Lithotomy b. Prone c. Left lateral decubitus d. Bending over the table while standing

ANS: C The nurse should place the female patient in the lithotomy position if the genitalia are being examined as well. The left lateral decubitus position is used for the rectal area alone. DIF: Cognitive Level: Understanding (Comprehension) REF: p. 725

6. A patient with a lack of oxygen to his heart will have pain in his chest and possibly in the shoulder, arms, or jaw. The nurse knows that the best explanation why this occurs is which one of these statements? a. A problem exists with the sensory cortex and its ability to discriminate the location. b. The lack of oxygen in his heart has resulted in decreased amount of oxygen to the areas experiencing the pain. c. The sensory cortex does not have the ability to localize pain in the heart; consequently, the pain is felt elsewhere. d. A lesion has developed in the dorsal root, which is preventing the sensation from being transmitted normally.

ANS: C The sensory cortex is arranged in a specific pattern, forming a corresponding map of the body. Pain in the right hand is perceived at a specific spot on the map. Some organs, such as the heart, liver, and spleen, are absent from the brain map. Pain originating in these organs is referred because no felt image exists in which to have pain. Pain is felt by proxy, that is, by another body part that does have a felt image. The other responses are not correct explanations. DIF: Cognitive Level: Understanding (Comprehension) REF: p. 636 MSC: Client Needs: Physiologic Integrity: Basic Care and Comfort

5. A 46-year-old man requires an assessment of his sigmoid colon. Which instrument or technique is most appropriate for this examination? a. Proctoscope b. Ultrasound c. Colonoscope d. Rectal examination with an examining finger

ANS: C The sigmoid colon is 40 cm long, and the nurse knows that it is accessible to examination only with the colonoscope. The other responses are not appropriate for an examination of the entire sigmoid colon. DIF: Cognitive Level: Understanding (Comprehension) REF: p. 722 MSC: Client Needs: Safe and Effective Care Environment: Management of Care

4. The area of the nervous system that is responsible for mediating reflexes is the: a. Medulla. b. Cerebellum. c. Spinal cord. d. Cerebral cortex.

ANS: C The spinal cord is the main highway for ascending and descending fiber tracts that connect the brain to the spinal nerves; it is responsible for mediating reflexes. DIF: Cognitive Level: Remembering (Knowledge) REF: p. 635 MSC: Client Needs: General

5. While gathering equipment after an injection, a nurse accidentally received a prick from an improperly capped needle. To interpret this sensation, which of these areas must be intact? a. Corticospinal tract, medulla, and basal ganglia b. Pyramidal tract, hypothalamus, and sensory cortex c. Lateral spinothalamic tract, thalamus, and sensory cortex d. Anterior spinothalamic tract, basal ganglia, and sensory cortex

ANS: C The spinothalamic tract contains sensory fibers that transmit the sensations of pain, temperature, and crude or light touch. Fibers carrying pain and temperature sensations ascend the lateral spinothalamic tract, whereas the sensations of crude touch form the anterior spinothalamic tract. At the thalamus, the fibers synapse with another sensory neuron, which carries the message to the sensory cortex for full interpretation. The other options are not correct. DIF: Cognitive Level: Applying (Application) REF: p. 635 MSC: Client Needs: General

25. An 18-year-old patient is having her first pelvic examination. Which action by the nurse is appropriate? a. Inviting her mother to be present during the examination b. Avoiding the lithotomy position for this first time because it can be uncomfortable and embarrassing c. Raising the head of the examination table and giving her a mirror so that she can view the examination d. Fully draping her, leaving the drape between her legs elevated to avoid embarrassing her with eye contact

ANS: C The techniques of the educational or mirror pelvic examination should be used. This is a routine examination with some modifications in attitude, position, and communication. First, the woman is considered an active participant, one who is interested in learning and in sharing decisions about her own health care. The woman props herself up on one elbow, or the head of the table is raised. Her other hand holds a mirror between her legs, above the examiner's hands. The young woman can see all that the examiner is doing and has a full view of her genitalia. The mirror works well for teaching normal anatomy and its relationship to sexual behavior. The examiner can ask her if she would like to have a family member, friend, or chaperone present for the examination. The drape should be pushed down between the patient's legs so that the nurse can see her face.

8. To palpate the temporomandibular joint, the nurse's fingers should be placed in the depression __________ of the ear. a. Distal to the helix b. Proximal to the helix c. Anterior to the tragus d. Posterior to the tragus

ANS: C The temporomandibular joint can be felt in the depression anterior to the tragus of the ear. The other locations are not correct. DIF: Cognitive Level: Understanding (Comprehension) REF: p. 578 MSC: Client Needs: Safe and Effective Care Environment: Management of Care

24. The nurse is performing an assessment on a 29-year-old woman who visits the clinic complaining of "always dropping things and falling down." While testing rapid alternating movements, the nurse notices that the woman is unable to pat both of her knees. Her response is extremely slow and she frequently misses. What should the nurse suspect? a. Vestibular disease b. Lesion of CN IX c. Dysfunction of the cerebellum d. Inability to understand directions

ANS: C When a person tries to perform rapid, alternating movements, responses that are slow, clumsy, and sloppy are indicative of cerebellar disease. The other responses are incorrect. DIF: Cognitive Level: Analyzing (Analysis) REF: p. 648 MSC: Client Needs: Physiologic Integrity: Physiologic Adaptation

32. A 30-year-old female patient is describing feelings of hopelessness and depression. She has attempted self-mutilation and has a history of suicide attempts. She describes difficulty sleeping at night and has lost 10 pounds in the past month. Which of these statements or questions is the nurse's best response in this situation? a. "Do you have a weapon?" b. "How do other people treat you?" c. "Are you feeling so hopeless that you feel like hurting yourself now?" d. "People often feel hopeless, but the feelings resolve within a few weeks."

ANS: C When the person expresses feelings of hopelessness, despair, or grief, assessing the risk of physical harm to him or herself is important. This process begins with more general questions. If the answers are affirmative, then the assessment continues with more specific questions. PTS: 1 DIF: Cognitive Level: Applying (Application) REF: p. 74 MSC: Client Needs: Psychosocial Integrity

40. A 46-year-old woman is in the clinic for her annual gynecologic examination. She voices a concern about ovarian cancer because her mother and sister died of it. Which statement does the nurse know to be correct regarding ovarian cancer? a. Ovarian cancer rarely has any symptoms. b. The Pap smear detects the presence of ovarian cancer. c. Women at high risk for ovarian cancer should have annual transvaginal ultrasonography for screening. d. Women over age 40 years should have a thorough pelvic examination every 3 years.

ANS: C With ovarian cancer, the patient may have abdominal pain, pelvic pain, increased abdominal size, bloating, and nonspecific gastrointestinal symptoms; or she may be asymptomatic. The Pap smear does not detect the presence of ovarian cancer. Annual transvaginal ultrasonography may detect ovarian cancer at an earlier stage in women who are at high risk for developing it. DIF: Cognitive Level: Applying (Application) REF: p. 772 MSC: Client Needs: Health Promotion and Maintenance

32. The nurse is testing superficial reflexes on an adult patient. When stroking up the lateral side of the sole and across the ball of the foot, the nurse notices the plantar flexion of the toes. How should the nurse document this finding? a. Positive Babinski sign b. Plantar reflex abnormal c. Plantar reflex present d. Plantar reflex 2+ on a scale from "0 to 4+"

ANS: C With the same instrument, the nurse should draw a light stroke up the lateral side of the sole of the foot and across the ball of the foot, similar to an upside-down J. The normal response is plantar flexion of the toes and sometimes of the entire foot. A positive Babinski sign is abnormal and occurs with the response of dorsiflexion of the big toe and fanning of all toes. The plantar reflex is not graded on a 0 to 4+ scale. DIF: Cognitive Level: Analyzing (Analysis) REF: p. 660 MSC: Client Needs: Safe and Effective Care Environment: Management of Care

39. While the nurse is taking the history of a 68-year-old patient who sustained a head injury 3 days earlier, he tells the nurse that he is on a cruise ship and is 30 years old. The nurse knows that this finding is indicative of a(n): a. Great sense of humor. b. Uncooperative behavior. c. Inability to understand questions. d. Decreased level of consciousness.

ANS: D A change in consciousness may be subtle. The nurse should notice any decreasing level of consciousness, disorientation, memory loss, uncooperative behavior, or even complacency in a previously combative person. The other responses are incorrect. DIF: Cognitive Level: Applying (Application) REF: p. 670 MSC: Client Needs: Physiologic Integrity: Physiologic Adaptation

37. A young swimmer comes to the sports clinic complaining of a very sore shoulder. He was running at the pool, slipped on some wet concrete, and tried to catch himself with his outstretched hand. He landed on his outstretched hand and has not been able to move his shoulder since. The nurse suspects: a. Joint effusion. b. Tear of rotator cuff. c. Adhesive capsulitis. d. Dislocated shoulder.

ANS: D A dislocated shoulder occurs with trauma involving abduction, extension, and external rotation (e.g., falling on an outstretched arm or diving into a pool). (See Table 22-2 for descriptions of the other conditions.) DIF: Cognitive Level: Analyzing (Analysis) REF: p. 621 MSC: Client Needs: Physiologic Integrity: Physiologic Adaptation

9. Which of these statements about the peripheral nervous system is correct? a. The CNs enter the brain through the spinal cord. b. Efferent fibers carry sensory input to the central nervous system through the spinal cord. c. The peripheral nerves are inside the central nervous system and carry impulses through their motor fibers. d. The peripheral nerves carry input to the central nervous system by afferent fibers and away from the central nervous system by efferent fibers.

ANS: D A nerve is a bundle of fibers outside of the central nervous system. The peripheral nerves carry input to the central nervous system by their sensory afferent fibers and deliver output from the central nervous system by their efferent fibers. The other responses are not related to the peripheral nervous system. DIF: Cognitive Level: Remembering (Knowledge) REF: p. 637 MSC: Client Needs: General

31. The nurse has completed the musculoskeletal examination of a patient's knee and has found a positive bulge sign. The nurse interprets this finding to indicate: a. Irregular bony margins. b. Soft-tissue swelling in the joint. c. Swelling from fluid in the epicondyle. d. Swelling from fluid in the suprapatellar pouch.

ANS: D A positive bulge sign confirms the presence of swelling caused by fluid in the suprapatellar pouch. The other options are not correct. DIF: Cognitive Level: Analyzing (Analysis) REF: p. 602 MSC: Client Needs: Physiologic Integrity: Physiologic Adaptation

12. The nurse is providing patient education for a man who has been diagnosed with a rotator cuff injury. The nurse knows that a rotator cuff injury involves the: a. Nucleus pulposus. b. Articular processes. c. Medial epicondyle. d. Glenohumeral joint.

ANS: D A rotator cuff injury involves the glenohumeral joint, which is enclosed by a group of four powerful muscles and tendons that support and stabilize it. The nucleus pulposus is located in the center of each intervertebral disk. The articular processes are projections in each vertebral disk that lock onto the next vertebra, thereby stabilizing the spinal column. The medial epicondyle is located at the elbow. DIF: Cognitive Level: Applying (Application) REF: p. 581 MSC: Client Needs: Physiologic Integrity: Physiologic Adaptation

23. When the nurse asks a 68-year-old patient to stand with his feet together and arms at his side with his eyes closed, he starts to sway and moves his feet farther apart. The nurse would document this finding as: a. Ataxia. b. Lack of coordination. c. Negative Homans sign. d. Positive Romberg sign.

ANS: D Abnormal findings for the Romberg test include swaying, falling, and a widening base of the feet to avoid falling. A positive Romberg sign is a loss of balance that is increased by the closing of the eyes. Ataxia is an uncoordinated or unsteady gait. Homans sign is used to test the legs for deep-vein thrombosis. DIF: Cognitive Level: Analyzing (Analysis) REF: p. 650 MSC: Client Needs: Health Promotion and Maintenance

4. When assessing aging adults, the nurse knows that one of the first things that should be assessed before making judgments about their mental status is: a. Presence of phobias b. General intelligence c. Presence of irrational thinking patterns d. Sensory-perceptive abilities

ANS: D Age-related changes in sensory perception can affect mental status. For example, vision loss (as detailed in Chapter 14) may result in apathy, social isolation, and depression. Hearing changes are common in older adults, which produces frustration, suspicion, and social isolation and makes the person appear confused. PTS: 1 DIF: Cognitive Level: Analyzing (Analysis) REF: p. 68 MSC: Client Needs: Psychosocial Integrity

14. A 70-year-old woman tells the nurse that every time she gets up in the morning or after she's been sitting, she gets "really dizzy" and feels like she is going to fall over. The nurse's best response would be: a. "Have you been extremely tired lately?" b. "You probably just need to drink more liquids." c. "I'll refer you for a complete neurologic examination." d. "You need to get up slowly when you've been lying down or sitting."

ANS: D Aging is accompanied by a progressive decrease in cerebral blood flow. In some people, this decrease causes dizziness and a loss of balance with a position change. These individuals need to be taught to get up slowly. The other responses are incorrect. DIF: Cognitive Level: Analyzing (Analysis) REF: p. 640 MSC: Client Needs: Health Promotion and Maintenance

24. A patient has been diagnosed with schizophrenia. During a recent interview, he shows the nurse a picture of a man holding a decapitated head. He describes this picture as horrifying but then laughs loudly at the content. This behavior is a display of: a. Confusion b. Ambivalence c. Depersonalization d. Inappropriate affect

ANS: D An inappropriate affect is an affect clearly discordant with the content of the person's speech. (See Table 5-5 for the definitions of the other terms.) PTS: 1 DIF: Cognitive Level: Analyzing (Analysis) REF: p. 81 MSC: Client Needs: Psychosocial Integrity

30. A 23-year-old patient in the clinic appears anxious. Her speech is rapid, and she is fidgety and in constant motion. Which of these questions or statements would be most appropriate for the nurse to use in this situation to assess attention span? a. "How do you usually feel? Is this normal behavior for you?" b. "I am going to say four words. In a few minutes, I will ask you to recall them." c. "Describe the meaning of the phrase, 'Looking through rose-colored glasses.'" d. "Pick up the pencil in your left hand, move it to your right hand, and place it on the table."

ANS: D Attention span is evaluated by assessing the individual's ability to concentrate and complete a thought or task without wandering. Giving a series of directions to follow is one method used to assess attention span. PTS: 1 DIF: Cognitive Level: Applying (Application) REF: p. 71 MSC: Client Needs: Psychosocial Integrity

47. A 35-year-old woman is at the clinic for a gynecologic examination. During the examination, she asks the nurse, "How often do I need to have this Pap test done?" Which reply by the nurse is correct? a. "It depends. Do you smoke?" b. "A Pap test needs to be performed annually until you are 65 years of age." c. "If you have two consecutive normal Pap tests, then you can wait 5 years between tests." d. "After age 30 years, if you have three consecutive normal Pap tests, then you may be screened every 2 to 3 years."

ANS: D Cervical cancer screening with the Pap test continues annually until age 30 years. After age 21, regardless of sexual history or activity, women should be screened every 3 years until age 30, then every 5 years until age 65. DIF: Cognitive Level: Applying (Application) REF: p. 741 MSC: Client Needs: Health Promotion and Maintenance

42. A 32-year-old woman tells the nurse that she has noticed "very sudden, jerky movements" mainly in her hands and arms. She says, "They seem to come and go, primarily when I am trying to do something. I haven't noticed them when I'm sleeping." This description suggests: a. Tics. b. Athetosis. c. Myoclonus. d. Chorea.

ANS: D Chorea is characterized by sudden, rapid, jerky, purposeless movements that involve the limbs, trunk, or face. Chorea occurs at irregular intervals, and the movements are all accentuated by voluntary actions. (See Table 23-5 for the descriptions of athetosis, myoclonus, and tics.) DIF: Cognitive Level: Analyzing (Analysis) REF: p. 680 MSC: Client Needs: Physiologic Integrity: Physiologic Adaptation

6. The nurse notices that a woman in an exercise class is unable to jump rope. The nurse is aware that to jump rope, one's shoulder has to be capable of: a. Inversion. b. Supination. c. Protraction. d. Circumduction.

ANS: D Circumduction is defined as moving the arm in a circle around the shoulder. The other options are not correct. DIF: Cognitive Level: Applying (Application) REF: p. 578 MSC: Client Needs: Physiologic Integrity: Physiologic Adaptation

43. When assessing muscle strength, the nurse observes that a patient has complete range of motion against gravity with full resistance. What grade of muscle strength should the nurse record using a 0- to 5-point scale? a. 2 b. 3 c. 4 d. 5

ANS: D Complete range of motion against gravity is normal muscle strength and is recorded as grade 5 muscle strength. The other options are not correct. DIF: Cognitive Level: Applying (Application) REF: p. 590 MSC: Client Needs: Physiologic Integrity: Physiologic Adaptation

26. A 20-year-old construction worker has been brought into the emergency department with heat stroke. He has delirium as a result of a fluid and electrolyte imbalance. For the mental status examination, the nurse should first assess the patient's: a. Affect and mood b. Memory and affect c. Language abilities d. Level of consciousness and cognitive abilities

ANS: D Delirium is a disturbance of consciousness (i.e., reduced clarity of awareness of the environment) with reduced ability to focus, sustain, or shift attention. Delirium is not an alteration in mood, affect, or language abilities. PTS: 1 DIF: Cognitive Level: Understanding (Comprehension) REF: p. 69 MSC: Client Needs: Psychosocial Integrity

5. Fibrous bands running directly from one bone to another that strengthen the joint and help prevent movement in undesirable directions are called: a. Bursa. b. Tendons. c. Cartilage. d. Ligaments.

ANS: D Fibrous bands running directly from one bone to another that strengthen the joint and help prevent movement in undesirable directions are called ligaments. The other options are not correct. DIF: Cognitive Level: Remembering (Knowledge) REF: p. 577 MSC: Client Needs: General

9. A 19-year-old woman comes to the clinic at the insistence of her brother. She is wearing black combat boots and a black lace nightgown over the top of her other clothes. Her hair is dyed pink with black streaks throughout. She has several pierced holes in her nares and ears and is wearing an earring through her eyebrow and heavy black makeup. The nurse concludes that: a. She probably does not have any problems. b. She is only trying to shock people and that her dress should be ignored. c. She has a manic syndrome because of her abnormal dress and grooming. d. More information should be gathered to decide whether her dress is appropriate.

ANS: D Grooming and hygiene should be noted—the person is clean and well groomed, hair is neat and clean, women have moderate or no makeup, and men are shaved or their beards or moustaches are well groomed. Care should be taken when interpreting clothing that is disheveled, bizarre, or in poor repair because these sometimes reflect the person's economic status or a deliberate fashion trend. PTS: 1 DIF: Cognitive Level: Applying (Application) REF: p. 70 MSC: Client Needs: Psychosocial Integrity

38. A 22-year-old woman is being seen at the clinic for problems with vulvar pain, dysuria, and fever. On physical examination, the nurse notices clusters of small, shallow vesicles with surrounding erythema on the labia. Inguinal lymphadenopathy present is also present. The most likely cause of these lesions is: a. Pediculosis pubis. b. Contact dermatitis. c. HPV. d. Herpes simplex virus type 2.

ANS: D Herpes simplex virus type 2 exhibits clusters of small, shallow vesicles with surrounding erythema that erupt on the genital areas. Inguinal lymphadenopathy is also present. The woman reports local pain, dysuria, and fever. (See Table 26-2 for more information and the descriptions of the other conditions.) DIF: Cognitive Level: Analyzing (Analysis) REF: p. 764 MSC: Client Needs: Physiologic Integrity: Physiologic Adaptation

30. In assessing a 70-year-old patient who has had a recent cerebrovascular accident, the nurse notices right-sided weakness. What might the nurse expect to find when testing his reflexes on the right side? a. Lack of reflexes b. Normal reflexes c. Diminished reflexes d. Hyperactive reflexes

ANS: D Hyperreflexia is the exaggerated reflex observed when the monosynaptic reflex arc is released from the influence of higher cortical levels. This response occurs with upper motor neuron lesions (e.g., a cerebrovascular accident). The other responses are incorrect. DIF: Cognitive Level: Applying (Application) REF: p. 655 MSC: Client Needs: Physiologic Integrity: Physiologic Adaptation

28. A 26-year-old woman was robbed and beaten a month ago. She is returning to the clinic today for a follow-up assessment. The nurse will want to ask her which one of these questions? a. "How are things going with the trial?" b. "How are things going with your job?" c. "Tell me about your recent engagement!" d. "Are you having any disturbing dreams?"

ANS: D In posttraumatic stress disorder, the person has been exposed to a traumatic event. The traumatic event is persistently reexperienced by recurrent and intrusive, distressing recollections of the event, including images, thoughts, or perceptions; recurrent distressing dreams of the event; and acting or feeling as if the traumatic event were recurring. PTS: 1 DIF: Cognitive Level: Applying (Application) REF: p. 82 MSC: Client Needs: Psychosocial Integrity

34. A 65-year-old woman is in the office for routine gynecologic care. She had a complete hysterectomy 3 months ago after cervical cancer was detected. Which statement does the nurse know to be true regarding this visit? a. Her cervical mucosa will be red and dry looking. b. She will not need to have a Pap smear performed. c. The nurse can expect to find that her uterus will be somewhat enlarged and her ovaries small and hard. d. The nurse should plan to lubricate the instruments and the examining hand adequately to avoid a painful examination.

ANS: D In the aging adult woman, natural lubrication is decreased; therefore, to avoid a painful examination, the nurse should take care to lubricate the instruments and the examining hand adequately. Menopause, with the resulting decrease in estrogen production, shows numerous physical changes. The cervix shrinks and looks pale and glistening. With the bimanual examination, the uterus feels smaller and firmer and the ovaries are not normally palpable. Women should continue cervical cancer screening up to age 65 years if they have an intact cervix and are in good health. Women who have had a total hysterectomy do not need cervical cancer screening if they have 3 consecutive negative Pap tests or 2 or more consecutive negative HIV and Pap tests within the last 10 years. DIF: Cognitive Level: Applying (Application) REF: p. 760 MSC: Client Needs: Health Promotion and Maintenance

11. The nurse is explaining to a patient that there are shock absorbers in his back to cushion the spine and to help it move. The nurse is referring to his: a. Vertebral column. b. Nucleus pulposus. c. Vertebral foramen. d. Intervertebral disks.

ANS: D Intervertebral disks are elastic fibrocartilaginous plates that cushion the spine similar to shock absorbers and help it move. The vertebral column is the spinal column itself. The nucleus pulposus is located in the center of each disk. The vertebral foramen is the channel, or opening, for the spinal cord in the vertebrae. DIF: Cognitive Level: Understanding (Comprehension) REF: p. 580 MSC: Client Needs: Physiologic Integrity: Physiologic Adaptation

17. The nurse is assessing orientation in a 79-year-old patient. Which of these responses would lead the nurse to conclude that this patient is oriented? a. "I know my name is John. I couldn't tell you where I am. I think it is 2010, though." b. "I know my name is John, but to tell you the truth, I get kind of confused about the date." c. "I know my name is John; I guess I'm at the hospital in Spokane. No, I don't know the date." d. "I know my name is John. I am at the hospital in Spokane. I couldn't tell you what date it is, but I know that it is February of a new year—2010."

ANS: D Many aging persons experience social isolation, loss of structure without a job, a change in residence, or some short-term memory loss. These factors affect orientation, and the person may not provide the precise date or complete name of the agency. You may consider aging persons oriented if they generally know where they are and the present period. They should be considered oriented to time if the year and month are correctly stated. Orientation to place is accepted with the correct identification of the type of setting (e.g., hospital) and the name of the town. PTS: 1 DIF: Cognitive Level: Applying (Application) REF: p. 76 MSC: Client Needs: Psychosocial Integrity

33. The nurse is examining a 3-month-old infant. While the nurse holds his or her thumbs on the infant's inner mid thighs and the fingers on the outside of the infant's hips, touching the greater trochanter, the nurse adducts the legs until the his or her thumbs touch and then abducts the legs until the infant's knees touch the table. The nurse does not notice any "clunking" sounds and is confident to record a: a. Positive Allis test. b. Negative Allis test. c. Positive Ortolani sign. d. Negative Ortolani sign.

ANS: D Normally, this maneuver feels smooth and has no sound. With a positive Ortolani sign, however, the nurse will feel and hear a "clunk," as the head of the femur pops back into place. A positive Ortolani sign also reflects hip instability. The Allis test also tests for hip dislocation but is performed by comparing leg lengths. DIF: Cognitive Level: Analyzing (Analysis) REF: p. 611 MSC: Client Needs: Physiologic Integrity: Physiologic Adaptation

18. A 22-year-old woman has been considering using oral contraceptives. As a part of her health history, the nurse should ask: a. "Do you have a history of heart murmurs?" b. "Will you be in a monogamous relationship?" c. "Have you carefully thought this choice through?" d. "If you smoke, how many cigarettes do you smoke per day?"

ANS: D Oral contraceptives, together with cigarette smoking, increase the risk for cardiovascular side effects. If cigarettes are used, then the nurse should assess the patient's smoking history. The other questions are not appropriate. DIF: Cognitive Level: Applying (Application) REF: p. 743 MSC: Client Needs: Physiologic Integrity: Pharmacologic and Parenteral Therapies

6. A woman who is 8 weeks pregnant is in the clinic for a checkup. The nurse reads on her chart that her cervix is softened and looks cyanotic. The nurse knows that the woman is exhibiting __________ sign and __________ sign. a. Tanner; Hegar b. Hegar; Goodell c. Chadwick; Hegar d. Goodell; Chadwick

ANS: D Shortly after the first missed menstrual period, the female genitalia show signs of the growing fetus. The cervix softens (Goodell sign) at 4 to 6 weeks, and the vaginal mucosa and cervix look cyanotic (Chadwick sign) at 8 to 12 weeks. These changes occur because of increased vascularity and edema of the cervix and hypertrophy and hyperplasia of the cervical glands. Hegar sign occurs when the isthmus of the uterus softens at 6 to 8 weeks. Tanner sign is not a correct response. DIF: Cognitive Level: Understanding (Comprehension) REF: p. 739 MSC: Client Needs: Health Promotion and Maintenance

38. A 68-year-old woman has come in for an assessment of her rheumatoid arthritis, and the nurse notices raised, firm, nontender nodules at the olecranon bursa and along the ulna. These nodules are most commonly diagnosed as: a. Epicondylitis. b. Gouty arthritis. c. Olecranon bursitis. d. Subcutaneous nodules.

ANS: D Subcutaneous nodules are raised, firm, and nontender and occur with rheumatoid arthritis in the olecranon bursa and along the extensor surface of the ulna. (See Table 22-3 for a description of the other conditions.) DIF: Cognitive Level: Analyzing (Analysis) REF: p. 623 MSC: Client Needs: Physiologic Integrity: Physiologic Adaptation

1. During a health history, a 22-year old woman asks, "Can I get that vaccine for human papilloma virus (HPV)? I have genital warts and I'd like them to go away!" What is the nurse's best response? a. "The HPV vaccine is for girls and women ages 9 to 26 years, so we can start that today." b. "This vaccine is only for girls who have not yet started to become sexually active." c. "Let's check with the physician to see if you are a candidate for this vaccine." d. "The vaccine cannot protect you if you already have an HPV infection."

ANS: D The HPV vaccine is appropriate for girls and women age 9 to 26 years and is administered to prevent cervical cancer by preventing HPV infections before girls become sexually active. However, it cannot protect the woman if an HPV infection is already present. DIF: Cognitive Level: Analyzing (Analysis) REF: p. 740 MSC: Client Needs: General

1. Which statement concerning the anal canal is true? The anal canal: a. Is approximately 2 cm long in the adult. b. Slants backward toward the sacrum. c. Contains hair and sebaceous glands. d. Is the outlet for the gastrointestinal tract.

ANS: D The anal canal is the outlet for the gastrointestinal tract and is approximately 3.8 cm long in the adult. It is lined with a modified skin that does not contain hair or sebaceous glands, and it slants forward toward the umbilicus. DIF: Cognitive Level: Remembering (Knowledge) REF: p. 721 MSC: Client Needs: General

14. While performing an assessment of the perianal area of a patient, the nurse notices that the pigmentation of anus is darker than the surrounding skin, the anal opening is closed, and a skin sac that is shiny and blue is noted. The patient mentioned that he has had pain with bowel movements and has occasionally noted some spots of blood. What would this assessment and history most likely indicate? a. Anal fistula b. Pilonidal cyst c. Rectal prolapse d. Thrombosed hemorrhoid

ANS: D The anus normally looks moist and hairless, with coarse folded skin that is more pigmented than the perianal skin, and the anal opening is tightly closed. The shiny blue skin sac indicates a thrombosed hemorrhoid. DIF: Cognitive Level: Analyzing (Analysis) REF: p. 726 MSC: Client Needs: Physiologic Integrity: Physiologic Adaptation

28. A professional tennis player comes into the clinic complaining of a sore elbow. The nurse will assess for tenderness at the: a. Olecranon bursa. b. Annular ligament. c. Base of the radius. d. Medial and lateral epicondyle.

ANS: D The epicondyles, the head of the radius, and the tendons are common sites of inflammation and local tenderness, commonly referred to as tennis elbow. The other locations are not affected. DIF: Cognitive Level: Analyzing (Analysis) REF: p. 595 MSC: Client Needs: Physiologic Integrity: Physiologic Adaptation

21. The nurse is testing the function of CN XI. Which statement best describes the response the nurse should expect if this nerve is intact? The patient: a. Demonstrates the ability to hear normal conversation. b. Sticks out the tongue midline without tremors or deviation. c. Follows an object with his or her eyes without nystagmus or strabismus. d. Moves the head and shoulders against resistance with equal strength.

ANS: D The following normal findings are expected when testing the spinal accessory nerve (CN XI): The patient's sternomastoid and trapezius muscles are equal in size; the person can forcibly rotate the head both ways against resistance applied to the side of the chin with equal strength; and the patient can shrug the shoulders against resistance with equal strength on both sides. Checking the patient's ability to hear normal conversation checks the function of CN VIII. Having the patient stick out the tongue checks the function of CN XII. Testing the eyes for nystagmus or strabismus is performed to check CNs III, IV, and VI. DIF: Cognitive Level: Applying (Application) REF: p. 646 MSC: Client Needs: Health Promotion and Maintenance

7. During the assessment of an 18-month-old infant, the mother expresses concern to the nurse about the infant's inability to toilet train. What would be the nurse's best response? a. "Some children are just more difficult to train, so I wouldn't worry about it yet." b. "Have you considered reading any of the books on toilet training? They can be very helpful." c. "This could mean that there is a problem in your baby's development. We'll watch her closely for the next few months." d. "The nerves that will allow your baby to have control over the passing of stools are not developed until at least 18 to 24 months of age."

ANS: D The infant passes stools by reflex. Voluntary control of the external anal sphincter cannot occur until the nerves supplying the area have become fully myelinated, usually around 1 to 2 years of age. Toilet training usually starts after the age of 2 years. DIF: Cognitive Level: Applying (Application) REF: p. 723 MSC: Client Needs: Health Promotion and Maintenance

16. The nurse is performing a mental status assessment on a 5-year-old girl. Her parents are undergoing a bitter divorce and are worried about the effect it is having on their daughter. Which action or statement might lead the nurse to be concerned about the girl's mental status? a. She clings to her mother whenever the nurse is in the room. b. She appears angry and will not make eye contact with the nurse. c. Her mother states that she has begun to ride a tricycle around their yard. d. Her mother states that her daughter prefers to play with toddlers instead of kids her own age while in daycare.

ANS: D The mental status assessment of infants and children covers behavioral, cognitive, and psychosocial development and examines how the child is coping with his or her environment. Essentially, the nurse should follow the same Association for Behavioral and Cognitive Therapies (ABCT) guidelines as those for the adult, with special consideration for developmental milestones. The best examination technique arises from a thorough knowledge of the developmental milestones (described in Chapter 2). Abnormalities are often problems of omission (e.g., the child does not achieve a milestone as expected). PTS: 1 DIF: Cognitive Level: Applying (Application) REF: p. 75 MSC: Client Needs: Psychosocial Integrity

4. When reviewing the musculoskeletal system, the nurse recalls that hematopoiesis takes place in the: a. Liver. b. Spleen. c. Kidneys. d. Bone marrow.

ANS: D The musculoskeletal system functions to encase and protect the inner vital organs, to support the body, to produce red blood cells in the bone marrow (hematopoiesis), and to store minerals. The other options are not correct. DIF: Cognitive Level: Remembering (Knowledge) REF: p. 577 MSC: Client Needs: General

25. During the taking of the health history of a 78-year-old man, his wife states that he occasionally has problems with short-term memory loss and confusion: "He can't even remember how to button his shirt." When assessing his sensory system, which action by the nurse is most appropriate? a. The nurse would not test the sensory system as part of the examination because the results would not be valid. b. The nurse would perform the tests, knowing that mental status does not affect sensory ability. c. The nurse would proceed with an explanation of each test, making certain that the wife understands. d. Before testing, the nurse would assess the patient's mental status and ability to follow directions.

ANS: D The nurse should ensure the validity of the sensory system testing by making certain that the patient is alert, cooperative, comfortable, and has an adequate attention span. Otherwise, the nurse may obtain misleading and invalid results. DIF: Cognitive Level: Analyzing (Analysis) REF: p. 644 MSC: Client Needs: Health Promotion and Maintenance

19. A 50-year-old woman is in the clinic for weakness in her left arm and leg that she has noticed for the past week. The nurse should perform which type of neurologic examination? a. Glasgow Coma Scale b. Neurologic recheck examination c. Screening neurologic examination d. Complete neurologic examination

ANS: D The nurse should perform a complete neurologic examination on an individual who has neurologic concerns (e.g., headache, weakness, loss of coordination) or who is showing signs of neurologic dysfunction. The Glasgow Coma Scale is used to define a person's level of consciousness. The neurologic recheck examination is appropriate for those who are demonstrating neurologic deficits. The screening neurologic examination is performed on seemingly well individuals who have no significant subjective findings from the health history. DIF: Cognitive Level: Applying (Application) REF: p. 644 MSC: Client Needs: Health Promotion and Maintenance

8. A 60-year-old man has just been told that he has benign prostatic hypertrophy (BPH). He has a friend who just died from cancer of the prostate. He is concerned this will happen to him. How should the nurse respond? a. "The swelling in your prostate is only temporary and will go away." b. "We will treat you with chemotherapy so we can control the cancer." c. "It would be very unusual for a man your age to have cancer of the prostate." d. "The enlargement of your prostate is caused by hormonal changes, and not cancer."

ANS: D The prostate gland commonly starts to enlarge during the middle adult years. BPH is present in 1 in 10 men at the age of 40 years and increases with age. It is believed that the hypertrophy is caused by hormonal imbalance that leads to the proliferation of benign adenomas. The other responses are not appropriate. DIF: Cognitive Level: Applying (Application) REF: p. 723 MSC: Client Needs: Health Promotion and Maintenance

44. During an assessment of a 32-year-old patient with a recent head injury, the nurse notices that the patient responds to pain by extending, adducting, and internally rotating his arms. His palms pronate, and his lower extremities extend with plantar flexion. Which statement concerning these findings is most accurate? This patient's response: a. Indicates a lesion of the cerebral cortex. b. Indicates a completely nonfunctional brainstem. c. Is normal and will go away in 24 to 48 hours. d. Is a very ominous sign and may indicate brainstem injury.

ANS: D These findings are all indicative of decerebrate rigidity, which is a very ominous condition and may indicate a brainstem injury. DIF: Cognitive Level: Analyzing (Analysis) REF: p. 688 MSC: Client Needs: Physiologic Integrity: Physiologic Adaptation

42. A 25-year-old woman comes to the emergency department with a sudden fever of 38.3° C and abdominal pain. Upon examination, the nurse notices that she has rigid, boardlike lower abdominal musculature. When the nurse tries to perform a vaginal examination, the patient has severe pain when the uterus and cervix are moved. The nurse knows that these signs and symptoms are suggestive of: a. Endometriosis. b. Uterine fibroids. c. Ectopic pregnancy. d. Pelvic inflammatory disease.

ANS: D These signs and symptoms are suggestive of acute pelvic inflammatory disease, also known as acute salpingitis (see Table 26-7). (For the descriptions of endometriosis and uterine fibroids, see Table 26-6; for a description of ectopic pregnancy, see Table 26-7.) DIF: Cognitive Level: Analyzing (Analysis) REF: p. 771 MSC: Client Needs: Physiologic Integrity: Physiologic Adaptation

20. A patient has had a cerebrovascular accident (stroke). He is trying very hard to communicate. He seems driven to speak and says, "I buy obie get spirding and take my train." What is the best description of this patient's problem? a. Global aphasia b. Broca's aphasia c. Echolalia d. Wernicke's aphasia

ANS: D This type of communication illustrates Wernicke's or receptive aphasia. The person can hear sounds and words but cannot relate them to previous experiences. Speech is fluent, effortless, and well articulated, but it has many paraphasias (word substitutions that are malformed or wrong) and neologisms (made-up words) and often lacks substantive words. Speech can be totally incomprehensible. Often, a great urge to speak is present. Repetition, reading, and writing also are impaired. Echolalia is an imitation or the repetition of another person's words or phrases. (See Table 5-4 for the definitions of the other disorders.) PTS: 1 DIF: Cognitive Level: Applying (Application) REF: p. 80 MSC: Client Needs: Psychosocial Integrity

49. A man who has had gout for several years comes to the clinic with a problem with his toe. On examination, the nurse notices the presence of hard, painless nodules over the great toe; one has burst open with a chalky discharge. This finding is known as: a. Callus. b. Plantar wart. c. Bunion. d. Tophi.

ANS: D Tophi are collections of monosodium urate crystals resulting from chronic gout in and around the joint that cause extreme swelling and joint deformity. They appear as hard, painless nodules (tophi) over the metatarsophalangeal joint of the first toe and they sometimes burst with a chalky discharge (see Table 22-6). (See Table 22-6 for descriptions of the other conditions.) DIF: Cognitive Level: Applying (Application) REF: p. 627 MSC: Client Needs: Physiologic Integrity: Physiologic Adaptation

46. The nurse is examining a 2-month-old infant and notices asymmetry of the infant's gluteal folds. The nurse should assess for other signs of what disorder? a. Fractured clavicle b. Down syndrome c. Spina bifida d. Hip dislocation

ANS: D Unequal gluteal folds may accompany hip dislocation after 2 to 3 months of age, but some asymmetry may occur in healthy children. Further assessment is needed. The other responses are not correct. DIF: Cognitive Level: Applying (Application) REF: p. 611 MSC: Client Needs: Safe and Effective Care Environment: Management of Care

15. Which of these individuals would the nurse consider at highest risk for a suicide attempt? a. Man who jokes about death b. Woman who, during a past episode of major depression, attempted suicide c. Adolescent who just broke up with her boyfriend and states that she would like to kill herself d. Older adult man who tells the nurse that he is going to "join his wife in heaven" tomorrow and plans to use a gun

ANS: D When the person expresses feelings of sadness, hopelessness, despair, or grief, assessing any possible risk of physical harm to him or herself is important. The interview should begin with more general questions. If the nurse hears affirmative answers, then he or she should continue with more specific questions. A precise suicide plan to take place in the next 24 to 48 hours with use of a lethal method constitutes high risk. PTS: 1 DIF: Cognitive Level: Applying (Application) REF: p. 74 MSC: Client Needs: Psychosocial Integrity

45. A 78-year-old man has a history of a cerebrovascular accident. The nurse notes that when he walks, his left arm is immobile against the body with flexion of the shoulder, elbow, wrist, and fingers and adduction of the shoulder. His left leg is stiff and extended and circumducts with each step. What type of gait disturbance is this individual experiencing? a. Scissors gait b. Cerebellar ataxia c. Parkinsonian gait d. Spastic hemiparesis

ANS: D With spastic hemiparesis, the arm is immobile against the body. Flexion of the shoulder, elbow, wrist, and fingers occurs, and adduction of the shoulder, which does not swing freely, is observed. The leg is stiff and extended and circumducts with each step. Causes of this type of gait include cerebrovascular accident. (See Table 23-6 for more information and for the descriptions of the other abnormal gaits.) DIF: Cognitive Level: Analyzing (Analysis) REF: p. 683 MSC: Client Needs: Physiologic Integrity: Physiologic Adaptation

1. During the assessment of deep tendon reflexes, the nurse finds that a patient's responses are bilaterally normal. What number is used to indicate normal deep tendon reflexes when the documenting this finding? ____+

ANS: 2 Responses to assessment of deep tendon reflexes are graded on a 4-point scale. A rating of 2+ indicates normal or average response. A rating of 0 indicates no response, and a rating of 4+ indicates very brisk, hyperactive response with clonus, which is indicative of disease. DIF: Cognitive Level: Applying (Application) REF: p. 655 MSC: Client Needs: Physiologic Integrity: Physiologic Adaptation


Ensembles d'études connexes

MICROM Lecture 11 chapter 13 (Viruses, Viroids, Prions)

View Set

What You / Others Like To Do and Other Useful Words

View Set

Geology 150 Test 4 Final (Glaciers, Wind, Desserts 21, 22, 23, anything about southern california geology & pictures and diagrams from last chapters, lithospheric keel, deepest part of the L.A. Basin, San Gabriel Anorthocite, L.A. Rivers, San Gabriel Mts.

View Set

Review of Chapter 1 Information: Negative & Positive Feedback

View Set

Cognitive Psychology- Practice Questions Ch. 10

View Set

Bio 1406 Lab 1 Collecting and Analyzing Data

View Set

Chapter 9 Toxicology Test Review Notes

View Set

FA Organ Systems Respiration- Pathology

View Set

ACCT 3303 Chapter 4, UTA Sargent

View Set

Ch. 22 diagnostic imaging CMA review

View Set

Organizational Behavior Chapters 5-8 Quizzes

View Set